Test Questions 2 Flashcards

1
Q

A teacher finds out that if she yells at a disruptive boy in class, he will calm down for a few minutes. Over
time, her yelling becomes louder and more frequent. Which of the following statements best describes, in
behavioral terms, what is happening in this situation?
A
The teacher’s yelling is serving as positive reinforcement for the boy, which is why he is becoming more and
more disruptive over time.
B
The teacher’s yelling is serving as positive punishment, but the boy is becoming habituated over time.
C
The teacher’s yelling is probably serving as negative punishment for her, because she likely finds the situation to
be aversive; however, it is serving as positive punishment for the boy, because he seems to enjoy being yelled at.
D
The teacher’s yelling is serving as positive reinforcement for herself, as she seems to enjoy yelling.

A

The Correct Answer is “B”
In behavioral terms, the term “positive” means that a stimulus is applied (not, as implied by choice C, that the
stimulus is pleasurable or enjoyable). In this case, the stimulus being applied is the teacher’s yelling. And
punishment means that the stimulus has the effect of reducing a behavior. In this case, the yelling has the effect
of temporarily reducing the boy’s disruptiveness, so it is punishment. The question illustrates a disadvantage of
punishment – the recipient becomes habituated (which just means he or she gets used to it) and it is necessary to
keep increasing the intensity of the punishment for it to be effective.

How well did you know this?
1
Not at all
2
3
4
5
Perfectly
2
Q

According to the research findings of Patterson and his colleagues, parents of aggressive children typically
use discipline which is:
A
consistent, but accompanied by humiliating verbal messages
B
consistent, but often followed by affection which sends mixed messages to the child
C
inconsistent, and often not associated with the child’s behavior
D
inconsistent, but which particularly ignores the child’s aggressive behaviors

A

The Correct Answer is “C”
Patterson found that parents of aggressive children often use harsh physical punishment which is applied
inconsistently and often not connected to the child’s behavior. Contrary to choice D, these parents tend to
reinforce aggressive behavior in their children with attention or approval.

How well did you know this?
1
Not at all
2
3
4
5
Perfectly
3
Q
3. Delirium may occur due to intoxication with all of the following substances except 
A
cocaine.
B
cannabis.
C
caffeine.
D
LSD.
A

c

How well did you know this?
1
Not at all
2
3
4
5
Perfectly
4
Q

An advantage of using a MANOVA over multiple one-way ANOVAs is that
A
the use of a MANOVA reduces the experiment-wise error rate.
B
a MANOVA can be used when the study involves more than one dependent variable.
C
a MANOVA is the more appropriate test when the researcher has an a priori hypotheses about the nature of the
relationship between the independent and dependent variables.
D
a MANOVA involves simpler mathematical calculations.

A

The Correct Answer is “A”
When a study involves two or more dependent variables, data can be analyzed with either multiple (one for each
dependent variable) statistical tests (e.g., multiple one-way ANOVAs) or one MANOVA. An advantage of the
latter technique is that it reduces the probability that at least one Type I error (incorrect rejection of the null
hypothesis) will be made. This is because the fewer statistical tests one conducts, the less likely it is that a Type I
error will occur. In an experiment that involves more than one comparison, the probability of at least one Type I
error is referred to as the experiment-wise error rate.

How well did you know this?
1
Not at all
2
3
4
5
Perfectly
5
Q
Children diagnosed with a Learning Disorder are most likely to receive a co-diagnosis of: 
A
Major Depressive Disorder
B
Tourette's Syndrome
C
Mental Retardation
D
Attention-Deficit/Hyperactivity Disorder
A

The Correct Answer is “D”
ADHD is the most frequent comorbid disorder, with about 20 to 25% of children with a Learning Disorder also
having ADHD.

How well did you know this?
1
Not at all
2
3
4
5
Perfectly
6
Q

Compared to decentralized networks, centralized networks
A
are less effective but more accepted by employees.
B
are more effective when the task is complex.
C
are more effective when the task is easy.
Dare more effective and more accepted by employees.

A

The Correct Answer is “C”
In a centralized communication network, all communication goes through one person (usually the person in
charge). This type of network is most effective when the problem or task is easy; i.e., it does not require a lot of
communication among workers to be resolved or accomplished. We can rule out A and D not only because these
networks are not automatically more or less effective than decentralized networks (again, it depends on the
nature of the task), but also because it is usually the leader, not the employees, who prefers a centralized
network.

How well did you know this?
1
Not at all
2
3
4
5
Perfectly
7
Q
Linehan's dialectical behavior therapy shows promise in the treatment of borderline patients. It involves a combination of treatment modalities that include 
A
inpatient therapy.
B
social skills training.
C
family therapy.
D
psychotropic medication.
A

The Correct Answer is “B”
Dialectical behavior therapy or DBT consists of outpatient individual therapy, social skills training and follow-up telephone consultations. It also discourages the expression of overt inappropriate affect. Its combination of social skills training (usually done in group therapy settings) and outpatient individual psychotherapy has been found to decrease hospitalization and suicide attempt rates (M. Linehan and C.A. Kehrer, Borderline Personality Disorder, in D.H. Barlow, Clinical Handbook of Psychological Disorders: A Step-by-Step Treatment Manual, New York, Guilford, 1993).

How well did you know this?
1
Not at all
2
3
4
5
Perfectly
8
Q
If you administer a measure of global job satisfaction to employees at the Acme Corporation, you would expect to find the highest levels of satisfaction among workers aged:
A
20 to 25.
B
30 to 35.
C
40 to 45.
D
50 to 55.
A

The Correct Answer is “D”
The relationship between age and satisfaction actually depends on what aspect of satisfaction you’re measuring. However, the question refers to global satisfaction, and the research has found that higher levels of global job satisfaction are associated with older age and vice versa.

How well did you know this?
1
Not at all
2
3
4
5
Perfectly
9
Q

You are completing a court-ordered evaluation in order to determine the person’s competency to stand trial and during the administration of the MMPI-2 your client jumps out of his seat and shouts “OK, OK, I killed her!” Now you are
A
obligated to include this information in your report; it is a court-ordered report.
B
obligated to include this information; privilege is waived because the mental status is part of the defense.
C
obligated to include this information; privilege is waived because you know have a Tarasoff situation.
D
not obligated to include the confessional information.

A

The Correct Answer is “D”
Since the question does not indicate that the client has waived consent, the law protects the defendant at this juncture. He is protected by laws that prohibit the use of evaluation data to determine the defendant’s guilt. The best course of action is to not include any incriminating evidence in the competency report, but instead only information relevant to the defendant’s competence (G.B. Melton, Psychological Evaluations for the Court, New York, Guilford, 1987).

How well did you know this?
1
Not at all
2
3
4
5
Perfectly
10
Q
Which of the following is least likely to cause secondary impotence?
A
medication use
B
alcohol use
C
diabetes mellitus
D
old age
A

The Correct Answer is “D”
Secondary impotence is diagnosed when a man persistently or recurrently fails to attain or maintain an erection even though in the past he has successfully achieved an erection. The erectile reflex is usually unimpaired in older men; therefore, aging alone is not likely to be a cause of impotence. The other responses are common physical causes of secondary impotence

How well did you know this?
1
Not at all
2
3
4
5
Perfectly
11
Q

An interpersonal therapist who is working with a client suffering from a prolonged grief reaction would attempt to
A
connect the person’s current grief reaction to losses of significant others in early childhood.
B
help the client understanding how the loss is impacting the current therapeutic relationship.
C
build up a positive relationship with the client and avoid discussing the loss until the termination phase of therapy.
D
modify the client’s maladaptive behaviors under the assumption that psychological change follows behavioral change.

A

The Correct Answer is “B”
Though “B” is an imprecise description of what an interpersonal therapist would do, it is the best answer available. Interpersonal therapists believe that depression is caused and maintained by disturbances in early life, especially attachment disturbances. Rather than focusing on the past, however, they focus on the connection between the presenting problem and client’s current relationships – including the current relationship with the therapist. Grief reactions are a common focus of interpersonal therapy.

How well did you know this?
1
Not at all
2
3
4
5
Perfectly
12
Q

Herbert Simon’s decision-making model suggests that decision makers
A
consider all alternatives and then pick the best one.
B
consider alternatives only until they find one that seems acceptable.
C
consider only those alternatives that have a 50% or better chance of success.
D
rely more on their affective than cognitive reactions when choosing an alternative.

A

The Correct Answer is “B”
To answer this question, you have to have the name Herbert Simon linked with the bounded rationality (administrative) model of decision making, which proposes that decision makers are not always completely rational in making choices. Instead, time and resources limit their consideration of alternatives, so they tend to consider alternatives only until a satisfactory one is identified.

How well did you know this?
1
Not at all
2
3
4
5
Perfectly
13
Q
Which of the following structures develops last?
A
cerebellum
B
frontal lobe
C
occipital lobe
D
temporal lobe
A

The Correct Answer is “B”
The brain develops in a predictable sequence from the least complex functions to the most complex. Although many areas of the brain are nearly fully developed at birth, the frontal lobe, which is responsible for higher-level thinking, motor behavior, and expressive language does not fully develop until young adulthood. The prefrontal cortex, located at the most anterior part of the frontal lobe, is responsible for most executive functions, and is the very last area of the brain to develop.

How well did you know this?
1
Not at all
2
3
4
5
Perfectly
14
Q
Release of the sex hormones by the gonads depends on signals from the
A
medulla
B
amygdala.
C
thalamus.
D
hypothalamus.
A

The Correct Answer is “D”
Once again, the hypothalamus is a “good guess” (and the correct response). One of the functions of the hypothalamus is to control the gonads through its influence on the pituitary gland.

How well did you know this?
1
Not at all
2
3
4
5
Perfectly
15
Q
Wernicke’s, Broca’s and conduction aphasia share which of the following difficulties?
A
repeating what is said
B
word prosody
C
reception
D
expression
A

The Correct Answer is “A”
A. Difficulty repeating words just spoken and recalling the name of familiar objects are characteristic of all three disorders. Conduction aphasia is due to damage to the nerve fibers that connect Broca’s to Wernicke’s area and the most typical result is difficulty repeating what one has heard. Wernicke’s, or receptive aphasia, primarily affects comprehension which results in impairment in spoken and written language, and anomia or problems recalling words. Broca’s aphasia is characterized by difficulty expressing language, including difficulty repeating what is said.

How well did you know this?
1
Not at all
2
3
4
5
Perfectly
16
Q

Which of the following contradicts the predictions of Herzberg’s two-factor theory?
A
Workers say the same work-related factors cause satisfaction and dissatisfaction with work.
B
Workers say they prefer leaders who show a high level of both instrumentality and consideration.
C
Job satisfaction and job performance have a reciprocal relationship.
D
Job enrichment increases satisfaction for both blue- and white-collar workers.

A

The Correct Answer is “A”
As long as you remembered that Herzberg views satisfaction and dissatisfaction as separate phenomena, you should have been able to pick the right answer to this question. According to Herzberg, the factors that cause dissatisfaction (hygiene factors) are different from those that contribute to satisfaction (motivator factors), which means that a person can be satisfied and dissatisfied at the same time.

How well did you know this?
1
Not at all
2
3
4
5
Perfectly
17
Q

Low LPC leaders would be most likely to say:
A
More beautiful paint colors in this office make us all feel better
B
Flexitime was a brillant invention
C
It’s time for a change around here
D
The first one to hand in this budget report wins the office lottery pool

A

The Correct Answer is “D”
According to Fiedler’s 1971 contingency theory, a leader’s effectiveness is determined by a combination of the leader’s style and the characteristics of the situation. Low LPC leaders are task and achievement oriented. Answers A and B could describe High LPC leaders who are primarily relationship oriented. Answer C could describe a transformational leader which Bass (1990) (see I/O section) described as change focused.

How well did you know this?
1
Not at all
2
3
4
5
Perfectly
18
Q

What effect do neurotransmitters have on postsynaptic cells?
A
an excitatory effect
B
an inhibitory effect
C
either an excitatory or inhibitory effect
D
the ability to decrease the action potential

A

The Correct Answer is “C”
Neurotransmitters can have an excitatory or inhibitory effect on postsynaptic cells, either increasing or decreasing the likelihood that an action potential will occur. Some neurotransmitters act as “neuromodulators” that increase or decrease the sensitivity of neurons to the effects of other neurotransmitters.

How well did you know this?
1
Not at all
2
3
4
5
Perfectly
19
Q
If data points are widely scattered around a regression line, it would indicate
A
high heteroscedasticity.
B
low heteroscedasticity.
C
low homoscedasticity.
D
a low correlation coefficient
A

The Correct Answer is “D”
Simply put, a lot of variance around the regression line indicates that the correlation isn’t too high. Be careful not to confuse this with the idea of heteroscedasticity. This term means that the scatter is uneven at different points of the continuum. For instance, there might be high variability around the regression line at low x (predictor) values, and low variability around the line at high x values. In other words, heteroscedasticity refers to a differential level of scatter, not high scatter.

How well did you know this?
1
Not at all
2
3
4
5
Perfectly
20
Q

Recent literature comparing leadership and management has identified several similarities and differences. Which of the following set of characteristics is more critical for today’s leaders than for managers?
A
rational, persistent, and tough-minded
B
problem-solving, persistent, and independent
C
visionary, authoritative, and tough-minded
D
independent, innovative, and flexible

A

The Correct Answer is “D”
A topic of recent interest in the I/O literature is the difference between leaders and managers. Characteristics that are considered particularly important for leaders include: visionary, creative, flexible, inspiring, courageous, and independent. See G. Capowski, Anatomy of a leader: Where are the leaders of tomorrow?, Management Review, 1994, p. 2.

How well did you know this?
1
Not at all
2
3
4
5
Perfectly
21
Q
A primary tenet of rational-emotive therapy is that:
A
a belief determines behavior.
B
self-attributions determine behavior.
C
conditioning determines behavior.
D
internalized role constructs determine behavior.
A

The Correct Answer is “A”
RET is basically a cognitively-based theory even though in practice therapists use many modalities. The idea is that beliefs – irrational beliefs – determine our maladaptive behaviors. The belief that you mustn’t make anyone mad at you is an example.

How well did you know this?
1
Not at all
2
3
4
5
Perfectly
22
Q
A child diagnosed with Attention -Deficit/Hyperactivity Disorder is most likely to also be diagnosed with:
A
Tourette's Disorder
B
Conduct Disorder
C
Enuresis
D
Obsessive-Compulsive Disorder
A

The Correct Answer is “B”
Attention-Deficit/Hyperactivity Disorder (ADHD) is highly comorbid with Conduct Disorder. Between 30% and 50% of children with ADHD also meet the criteria for Conduct Disorder, with the highest comorbidity rates among the two subtypes marked by hyperactivity-impulsivity (Hyperactive-Impulsive and Combined Types). The percentage of patients with Conduct Disorder who also have ADHD is nearly 70%. (T. P. Beauchaine, E. S. Katkin, Z. Strassberg, & J. Snarr. Disinhibitory psychopathology in male adolescents: Discriminating Conduct Disorder from Attention-Deficit/ Hyperactivity Disorder through concurrent assessment of multiple autonomic states. Journal of Abnormal Psychology, 2001, 110(4), 610-624). If you incorrectly selected Tourette’s Disorder (A), you may have been thinking of the reverse relationship. That is, among those with Tourette’s Disorder the comorbidity of ADHD is at least 50%; however, most patients with ADHD do not have Tourette’s Disorder.

How well did you know this?
1
Not at all
2
3
4
5
Perfectly
23
Q
Communication-interaction therapy espouses that communication has both a "report" function and a
A
Principle of equifinality
B
Command function
C
Circular model of causality
D
Paradoxical strategy
A

The Correct Answer is “B”
Family therapists from the Mental Research Institute in Palo Alto such as Gregory Bateson, Virginia Satir, and Jay Haley described communication as having a “report function” that contains the content or informational aspect of the communication, and the “command function”, that is often conveyed nonverbally and exemplifies the relationship between the communicators. The other choices are other concepts from the Mental Research Institute. “Principles of equifinality” refers to the idea that no matter where the system change occurs, the end result is the same. “Circular model of causality” is a concept in their approach that describes a symptom as both a cause and an effect of dysfunctional communication patterns. “Paradoxical strategies” include prescribing the symptom and relabeling, or changing the label a family attaches to a problem in order to change the meaning.

How well did you know this?
1
Not at all
2
3
4
5
Perfectly
24
Q
Depth perception in infants develops in which of the following sequences?
A
kinetic, pictorial, binocular
B
pictorial, binocular, kinetic
C
kinetic, binocular, pictorial
D
binocular, kinetic, pictorial
A

The Correct Answer is “C”
Research has indicated that infants develop depth perception in a predictable sequence: kinetic, binocular, and pictorial. Kinetic depth cues are based on movements of objects in the environment or the body. Babies as young as 3 weeks begin perceiving kinetic cues. For example, they will blink their eyes defensively when an object approaches their face and looks as if its going to hit them. Binocular depth perception begins between 2 and 3 months. And pictorial depth cues, which are used by artists to make paintings look three-dimensional, begins to develop at about 7 months of age.

How well did you know this?
1
Not at all
2
3
4
5
Perfectly
25
Q
An organization decides to implement changes based on a job characteristics assessment. Which of the following is least likely to be affected?
A
work quality
B
motivation
C
satisfaction
D
absenteeism
A

The Correct Answer is “A”
Studies show when jobs are redesigned according to the job characteristics model there are improvements in satisfaction, absenteeism, turnover, and motivation. Work quality is less likely to be affected.

How well did you know this?
1
Not at all
2
3
4
5
Perfectly
26
Q
Which of the following is an application of negative punishment?
A
D.R.O.
B
overcorrection
C
Premack Principle
D
time-out
A

The Correct Answer is “D”
Negative punishment is used to decrease a behavior by taking away a stimulus. In time-out, undesirable behaviors are reduced by taking away opportunities for positive reinforcement for a specified period of time.

How well did you know this?
1
Not at all
2
3
4
5
Perfectly
27
Q
The “fourth force” in psychology has been identified as:
A
humanistic
B
psychodynamic
C
multiculturalism
D
behavioral
A

The Correct Answer is “C”
C. Multiculturalism has been defined as the fourth force in psychology, one which complements the behavioral, psychodynamic and humanistic explanations of human behavior. This “fourth force” originated within the civil rights and social movements of the 1960s. Pedersen (1991) defined multiculturalism as “a wide range of multiple groups without grading, comparing, or ranking them as better or worse than one another and without denying the very distinct and complementary or even contradictory perspectives that each group brings with it.” Others advocate a narrower definition of multiculturalism, with attention to the racial/ethnic minority groups within a culture. Multicultural counseling often includes issues of race, ethnicity, culture, social class, sexual orientation, gender, physical ability, age, and religious preference. (See: Pedersen, Paul B. (1991). Multiculturalism as a fourth force in counseling. Journal of Counseling and Development, 70, 6-12.)

How well did you know this?
1
Not at all
2
3
4
5
Perfectly
28
Q

Tricyclic antidepressants, as compared to MAO inhibitors,
A
are more effective in the treatment of the vegetative symptoms of depression.
B
are more effective in the treatment of atypical depressions.
C
are less effective in the treatment of panic disorder.
D
are associated with a greater range of dangerous side effects.

A

The Correct Answer is “A”
The tricyclic antidepressants include imipramine, clomimpramine, and amitriptyline. In the treatment of depression, they are most effective in relieving vegetative symptoms such as appetitive, sleep, and motor disturbances. By contrast, the MAO inhibitors are more effective in the treatment of atypical depressions.

How well did you know this?
1
Not at all
2
3
4
5
Perfectly
29
Q
29. If a job selection test has lower validity for Hispanics as compared to White or African-Americans, you could say that ethnicity is acting as a:
A
confounding variable
B
criterion contaminator
C
discriminant variable
D
moderator variable
A

The Correct Answer is “D”
A moderator variable is any variable which moderates, or influences, the relationship between two other variables. If the validity of a job selection test is different for different ethnic groups (i.e. there is differential validity), then ethnicity would be considered a moderator variable since it is influencing the relationship between the test (predictor) and actual job performance (the criterion). A confounding variable (A) is a variable in a research study which is not of interest to the researcher, but which exerts a systematic effect on the DV. Criterion contamination (B) is the artificial inflation of validity which can occur when raters subjectively score ratees on a criterion measure after they have been informed how the ratees scored on the predictor.

How well did you know this?
1
Not at all
2
3
4
5
Perfectly
30
Q

According to research on parenting styles, which of the following describes the parents who are most likely to raise very aggressive children?
A
attentive parents who are very controlling of their children’s behavior
B
parents who use frequent and intermittent violence and are very controlling of their children’s behavior
C
loving parents with a laissez-faire attitude toward their children’s behavior
D
parents who use frequent and intermittent violence and have a laissez-faire attitude toward their children’s behavior

A

The Correct Answer is “D”
Researchers in the 1950s identified two dimensions of parenting styles: permissiveness and affection. Permissiveness is a continuum that ranges from autonomy on one extreme to control at the other; affection is a continuum that ranges from hostility to love. Parents who combine hostility with autonomy (e.g., those who combine violent discipline with a laissez-faire attitude toward their children) are likely to produce disobedient and aggressive children.

How well did you know this?
1
Not at all
2
3
4
5
Perfectly
31
Q
Journal reviewers who show strong bias against manuscripts that report results contrary to their theoretical perspective are demonstrating:
A
self-serving bias
B
confirmatory bias
C
fundamental attribution bias
D
self verification theory
A
The Correct Answer is "B"
"Confirmatory bias" or "confirmation bias" is the tendency to seek, interpret, and create information that verifies our existing beliefs. Self-serving bias (A) is the tendency to attribute one's successes to internal factors and one's failures to external factors. The fundamental attribution bias or error (C) refers to the tendency to overestimate dispositional (personality) factors and underestimate situational factors in explaining a person's behavior. Self
verification theory (D) proposes that people seek confirmation of their self-concept.
How well did you know this?
1
Not at all
2
3
4
5
Perfectly
32
Q
The "Big Five" personality factors were based on which of the following approaches?
A
cognitive-behavioral
B
industrial-organizational
C
classical test theory
D
lexical
A

This is one of those obscure questions which could appear on your exam. A lexical approach was first used by Sir Francis Galton in the 1880’s. “Lexical” refers to the use of a dictionary, in this case, to identify personality traits. It is an atheoretical approach and was used to identify the Big Five Personality traits by performing a factor analysis on all the personality traits found in the dictionary. The Big Five traits are: conscientiousness, extroversion, agreeableness, openness, and emotional stability/neuroticism. These five traits are believed to underlie all other personality characteristics.

How well did you know this?
1
Not at all
2
3
4
5
Perfectly
33
Q

You are trying to recall the facts surrounding a mugging you recently witnessed. According to the notion of state dependence, your recall of these facts will be best if you are in the same __________ when you recall the crime as you were at the time you witnessed it.

A
attitudinal state.
B
physical environment.
C
emotional state.
D
state of alertness.
A

The Correct Answer is “C”
The notion of state dependence predicts that memory is maximized when we are in the same emotional state during learning and recall.

How well did you know this?
1
Not at all
2
3
4
5
Perfectly
34
Q

The research suggests that, to control excessive aggression in children, the best approach is:
A
time-out and similar behavioral techniques.
B
opportunities for catharsis.
C
explaining the consequences of aggressive acts.
D
social-skills training.

A

The Correct Answer is “D”
A number of techniques have been found useful for reducing aggression in children, but many of them (e.g., catharsis) have only short-term effects. In the long run, the best thing to do is to teach aggressive children alternative, nonaggressive, prosocial behaviors, which is a component of social-skills training.

How well did you know this?
1
Not at all
2
3
4
5
Perfectly
35
Q

Coding Personality Disorders on a separate axis, according to the DSM-IV-TR, is due to:
A
the need to reduce the possibility of being overlooked
B
the need to reduce the possibility of prioritization over an Axis I disorder
C
having a differing etiology than Axis I disorders
D
having less daily functioning interference than Axis I disorders

A

The Correct Answer is “A”
A. In the DSM-IV-TR’s description of Axis II, Personality Disorders and Mental Retardation are classified on a separate axis as they “might otherwise be overlooked when attention is directed to the usually more florid Axis I disorders” and does not suggest “pathogenesis or range of appropriate treatment is fundamentally different from that for the disorders coded on Axis I.”

How well did you know this?
1
Not at all
2
3
4
5
Perfectly
36
Q

Research comparing the relationship between age and therapy outcome has produced mixed results. However, in their most recent meta-analysis of the research, Weisz and his colleagues (1995) found:
A
no relationship between age and outcome.
B
that therapy is more beneficial for children and adolescents than adults.
C
that therapy is more beneficial for children than adolescents, especially among girls.
D
that therapy is more beneficial for adolescents than children, especially among girls.

A

The Correct Answer is “D”
Earlier meta-analyses of the outcome studies for children and adolescents found either no difference for adolescents and children or a slight superiority for children. In contrast, the more recent Weisz et al. meta-analysis found that therapy has better outcomes for adolescents than children, especially female adolescents and when the counselor is a professional or student (versus paraprofessional). Note that Weisz et al. didn’t compare outcomes for children and adolescents to outcomes for adults, but other meta-analyses suggest that adults do somewhat better.

How well did you know this?
1
Not at all
2
3
4
5
Perfectly
37
Q

You receive a subpoena for a current client’s therapy records from a process server hired by the court. This means that you must:
A
inform the server that you cannot comply with the subpoena because therapy records are privileged.
B
tell the server to wait while you make a copy of the records.
C
appear at a designated place and time with a copy of the records.
D
leave a copy of the records at the courthouse within 48 hours.

A

The Correct Answer is “C”
A subpoena to produce records or documents (known as a subpoena duces tecum) requires a person to appear at a designated time and place with a copy of the records. It does not necessarily require the person to release those records; this is a matter for the court to decide, following a hearing on privilege, if the issue of privilege is being contested. If your client has waived the privilege, however, you would release the records to the court at the time of your appearance.

How well did you know this?
1
Not at all
2
3
4
5
Perfectly
38
Q
An MRI of a patient in the early stages of Huntington's disease would indicate:
A
no abnormalities.
B
overactivity in the temporal lobes.
C
reduced volume of the basal ganglia.
D
reduced volume of the ventricles.
A

The Correct Answer is “C”
To answer this question, you need to know that Huntington’s disease is due to degeneration in several areas of the brain including the basal ganglia and that damage can often be detected by an MRI or other brain imaging technique even before the person exhibits symptoms.

How well did you know this?
1
Not at all
2
3
4
5
Perfectly
39
Q
Hypnosis involves three factors
A
Absorption, regression, dissociation
B
Dissociation, absorption, suggestibility
C
Suggestibility, dissociation, regression
D
Regression, distortion, suggestibility
A

The Correct Answer is “B”
According to Hales, Yudofsky and Talbott, (1944), hypnosis involves three factors, 1) absorption, whereby the individual is completely engrossed in a central experience, 2) dissociation, whereby the ordinary functioning of consciousness and memory are altered in some way and 3) suggestibility, whereby individuals have a tendency to be less inhibited and restricted while in the trance-like state.

How well did you know this?
1
Not at all
2
3
4
5
Perfectly
40
Q

In the context of expectancy theory, instrumentality refers to:
A
the willingness of a worker to exert effort.
B
the likelihood of effort leading to performance
C
the likelihood of performance leading to certain outcomes.
D
the value of the outcomes of performance.

A

The Correct Answer is “C”
C. Expectancy theory is based on the premise that motivation is a cognitive process involving three variables: expectancy, instrumentality and valence. Expectancy refers to the belief that effort will lead to success performance. Instrumentality refers to the beliefs that successful performance will result in certain outcomes; the value placed on the outcomes of performance is referred to as valence.

How well did you know this?
1
Not at all
2
3
4
5
Perfectly
41
Q
The WISC-III subtests that are most sensitive to perceptual-motor problems are:
A
block design and object assembly.
B
object assembly and picture completion.
C
digit span and coding.
D
picture arrangement and picture completion.
A

The Correct Answer is “A”
Of the choices listed, the block design and object assembly subtests are the most direct measures of perceptual and motor skills.

How well did you know this?
1
Not at all
2
3
4
5
Perfectly
42
Q
A parent finds that she has to nag her son more and more to get him to stop teasing his sister. The last time it happened, the boy finally stopped his teasing after his mother's 16th angry request. The boy's compliance is best described as the result of
A
escape conditioning.
B
avoidance conditioning.
C
stimulus discrimination.
D
stimulus generalization.
A

The Correct Answer is “A”
In this situation, the boy stops teasing his sister because, presumably, this stops his mother’s nagging. In other words, by complying with her request, he escapes an aversive stimulus.

How well did you know this?
1
Not at all
2
3
4
5
Perfectly
43
Q

According to family therapists:
A
when one family member improves, others will improve also.
B
no member of a family will improve unless all members improve.
C
if the identified patient improves, the family system will improve.
D
if the family system changes, the identified patient will improve.

A

The Correct Answer is “D”
You needed to approach this question from the point of view of a system. A system means that everything functions together. Knowing just that little piece, you probably could have gotten to the correct answer: once the system changes, the individuals will change. The work is at the systems level, not at the individual level. The alternative stating that “no member will improve unless all members improve” comes close to this idea, but it’s not exactly the point of system theory. And, besides, you don’t have to have all members improving before any one member improves. According to systems theory, individual symptoms can remit, however, without systemic change, other problems (either in that individual or in another family member) will arise.

How well did you know this?
1
Not at all
2
3
4
5
Perfectly
44
Q
Which of the following stages in Erikson's theory of psychosocial development corresponds to Freud's latency stage?
A
trust versus mistrust
B
autonomy versus shame
C
initiative versus guilt
D
industry versus inferiority
A

The Correct Answer is “D”
Erikson’s industry vs. inferiority stage occurs approximately between the ages of 6-11. Freud’s latency stage occurs at about the same time.

How well did you know this?
1
Not at all
2
3
4
5
Perfectly
45
Q
In late childhood and adolescence, sibling relationships tend to become egalitarian; however, during middle
childhood they can best be described as:
A
distant and distrustful
B
conflicted and close
C
abusive and uncaring
D
dominant and submissive
A

The Correct Answer is “B”
During middle childhood sibling relationships are characterized by a combination of conflict and closeness. During this period siblings typically fight and experience friction, yet also report having greater warmth and companionship with each other.

How well did you know this?
1
Not at all
2
3
4
5
Perfectly
46
Q
All of the following are generally considered to be characteristic of brief psychotherapy, except:
A
time limited therapy
B
lack of theoretical orientation
C
effective for acute symptoms
D
encourages the development of a positive transference between the client and the therapist.
A

The Correct Answer is “B”
There are many different approaches to brief therapy including psychodynamic, cognitive-behavioral, and eclectic to name a few; however, brief therapy is always based on theory. Brief therapy, as its name implies, is time limited and effective for acute symptoms. A positve transferance is also generally considered to be essential to the success of brief therapy.

How well did you know this?
1
Not at all
2
3
4
5
Perfectly
47
Q
The variable most directly associated with a need for personal space is
A
education.
B
sex.
C
income.
D
cultural background.
A

The Correct Answer is “D”
Although some studies suggest that cultural differences in the need for personal space are not as great as once believed, this is still the best answer. For instance, research has illustrated that adult Americans generally require more personal space than adults in other cultures.

How well did you know this?
1
Not at all
2
3
4
5
Perfectly
48
Q

The suicide rate for African-American adolescent males over the past few decades has been:
A
higher than the rate for white adolescent males, but there has recently been an increase in the rates for both groups.
B
lower than the rate for white adolescents males, but there has been an increase in the rates for both groups.
C
higher than the rate for white adolescent males, but there has recently been a decrease in the rates for both groups.
D
lower than the rate for white adolescent males, but there has been a decrease in the rates for both groups.

A

The Correct Answer is “B”
Across all age groups and both genders, the suicide rate is higher for whites than for African-Americans. Of all demographic groups, the one associated with the highest increase in suicide rate over the past few decades is young white male – the rate has almost tripled. The rate has also gone up among African-American males, but
not by as much. And here’s an additional piece of information that’s not directly related to this question: In African-Americans, suicide occurs at the highest rate among individuals between the ages of 20-34; among whites, the highest rate of suicide occurs near the end of the life cycle.

How well did you know this?
1
Not at all
2
3
4
5
Perfectly
49
Q

Jerome and Jamal have significant problems with their peers. However, while Jerome is rejected by his classmates, Jamal is neglected. If Jerome and Jamal change schools, which of the following is most likely to happen?
A
Jerome may be better accepted by his new classmates but Jamal will continue to be neglected.
B
Jamal may be better accepted by his new classmates but Jerome will continue to be rejected.
C
Jerome and Jamal will both be better accepted by their new classmates.
D
Jerome and Jamal will both continue to have the same peer problems they had in their old school.

A

The Correct Answer is “B”
Studies looking at the outcomes for rejected and neglected children have found that rejection is more stable than neglect. For example, when rejected and neglected change schools, neglected children may experience improvements in their peer status, while rejected children continue to be rejected by the new peer group.

How well did you know this?
1
Not at all
2
3
4
5
Perfectly
50
Q
A person with high self-esteem is likely to make what type of attributions for his or her achievements?
A
internal and specific
B
internal and global
C
external and specific
D
external and global
A

The Correct Answer is “A”
This question is related to Weiner’s attributional theory of motivation and emotion. Attributions are our causal explanations for events and the behavior of ourselves and others. According to Weiner, attributions can be due to factors that are either internal or external, stable or unstable, controllable or uncontrollable, intentional or unintentional, and global or specific. Someone with high self-esteem would take credit for his or her achievements and be proud of them. Thus, he or she would likely attribute them to internal, stable, controllable, intentional, and specific causes. The attributions are most likely specific rather than global because a person with high self-esteem will acknowledge that he or she isn’t successful in everything. For example, a very successful athlete or CEO would not assume proficiency in brain surgery.

How well did you know this?
1
Not at all
2
3
4
5
Perfectly
51
Q
The ability to remember the first and last parts of a lecture, but not the middle part, would most likely be due to:
A
retroactive inhibition
B
primacy and recency effects
C
method of loci
D
selective attention
A

The Correct Answer is “B”
The ability to recall information that is presented at the beginning and ending of a list (or in this case, a lecture) better than the information presented in the middle is known, respectively, as the primacy and recency effects. Together they are referred to as the serial position effect. The serial position effect is believed to occur because material presented at the beginning is transferred to long-term memory, while information at the end is still in short-term memory. Whereas, information in the middle can be affected by interference from information that came before and afterwards, preventing storage in short- or long-term memory. Retroactive inhibition (A) occurs when a new experience interferes with the recall of an earlier one. Method of loci (C) is a mnemonic device in which items are associated with mental images of places. Selective attention (D) involves focusing on one event while filtering out or ignoring irrelevant events.

How well did you know this?
1
Not at all
2
3
4
5
Perfectly
52
Q

If one were to apply the confluence model to predict the intellectual and achievement scores of children, the most accurate prediction would be that
A
middle children would show the highest score when compared to both earlier and later born children.
B
children from small families would score the highest.
C
only-borns would be the highest scoring group.
D
later borns would be the highest scoring group.

A

The Correct Answer is “C”
The confluence model states simply that each succeeding child has less of the family’s resources available to him or her. The first child doesn’t have to share with anyone. The next child shares with one older sibling. The next child must share with two older siblings. The theory has been tested and found to predict accurately: Only-borns and children from small families tend to, on the average, do better on measures of intellect and achievement than later borns. Note that the choice about only-borns having the highest intelligence is a better answer than the choice about children from a small family. This is because, from the perspective of the confluence model, only-borns have even more of the family’s resources available to them than children from a small family.

How well did you know this?
1
Not at all
2
3
4
5
Perfectly
53
Q
Primary memory is the same as
A
implicit memory.
B
short-term memory.
C
procedural memory.
D
episodic memory.
A

The Correct Answer is “B”
Primary memory is another term for short-term memory. On the other hand, the terms secondary and long-term memory are also often used interchangeably. Implicit memory (answer A) is an unconscious, nonintentional form of memory. Procedural memory (answer C) is memory of how to do certain activities, for example, driving a car, while episodic memory (answer D) is the memory of a particular episode, such as your last birthday.

How well did you know this?
1
Not at all
2
3
4
5
Perfectly
54
Q

Which of the following is true regarding Obsessive-Compulsive Disorder (OCD) and Obsessive Compulsive Personality Disorder (OCPD)?
A
Both OCD and OCPD are characterized by the presence of true obsessions and/or compulsions.
B
Unlike OCD, OCPD involves a preoccupation with orderliness, perfectionism, and control.
C
Obsessions and compulsions are less severe in OCPD.
D
Unlike OCPD, the DSM-IV-TR criteria for OCD requires onset of symptoms before age 18.

A

The Correct Answer is “B”
Only Obsessive-Compulsive Disorder (OCD) requires the presence of obsessions and/or compulsions. The diagnosis “Obsessive-Compulsive Personality Disorder” (OCPD) is somewhat of a misnomer in that it is not characterized by either obsessions or compulsions. Rather, OCPD involves a preoccupation with orderliness, perfectionism, and control. Contrary to Choice “D,” there are no age requirements for either diagnosis.

How well did you know this?
1
Not at all
2
3
4
5
Perfectly
55
Q

According to Lenore Walker, which of the following best describes the dynamic that keeps battered women “hooked” into their relationship with the batterer?
A
Due to having grown up in an abusive family, the victim believes that battering is a normal part of relationships.
B
The costs of the abuse and the benefits of remaining in the relationship are about equal.
C
The woman fears that the abuse will become even more severe if she tries to leave the relationship.
D
The woman lacks knowledge about the resources available to help her leave the relationship.

A

The Correct Answer is “B”
Lenore Walker describes a cycle of violence that involves three stags: tension building, acute battering incident, and loving contrition. According to Walker, most of the benefits of the relationship occur in the third stage, when the batterer offers apologies, assurances that the attacks will never happen again, and declarations of love. The relationship tends to remain stable when the balance between the costs of the abuse and the benefits of the relationship are fairly similar. As violence escalates, the relationship becomes more unstable, and the man escalates his charming behavior in an attempt to restore stability.

How well did you know this?
1
Not at all
2
3
4
5
Perfectly
56
Q

According to Hersey and Blanchard (1974), a “telling” leader is most effective when employees are low in both ability and willingness to assume responsibility. A telling leadership style is characterized by a
A
low task and low relationship orientation.
B
low task and high relationship orientation.
C
high task and high relationship orientation.
D
high task and low relationship orientation.

A

The Correct Answer is “D”
The different leadership styles defined by Hersey and Blanchard involve different combinations of task and relationship orientation. A high task, low relationship orientation defines the telling leadership style. Answer A is a delegating leader; B is a participating leader, and C is a selling leader.

How well did you know this?
1
Not at all
2
3
4
5
Perfectly
57
Q

At the request of an attorney, a psychologist in a rural community agrees to conduct a psychological evaluation and provide treatment for the same person. This would probably be considered:
A
ethical, since both services are within the boundaries of a professional relationship
B
ethical, if the psychologist takes reasonable steps to minimize any negative effects
C
unethical, since evaluation and treatment are inherently conflicting
D
unethical, since only a patient, not an attorney can request treatment services

A

The Correct Answer is “B”
As a general rule, psychologists should avoid potentially conflicting relationships, which could result when conducting both an evaluation and treatment for a patient. However, according to APA’s Specialty Guidelines for Forensic Psychologists (1991), “When it is necessary to provide both evaluation and treatment services to a party in a legal proceeding (as may be the case in small forensic hospital settings or small communities), the forensic psychologist takes reasonable steps to minimize the potential negative effects of these circumstances on the rights of the party, confidentiality, and the process of treatment and evaluation.” [Law and Human Behavior, 15 (6), p.659].

How well did you know this?
1
Not at all
2
3
4
5
Perfectly
58
Q

A number of books in the popular press have been written regarding the relationship between psychological factors and cancer. Which of the following statements best reflects the outcome of scientific studies of this issue?
A
Psychological factors are related both to the onset of cancer and the success of recovery from it.
B
Psychological factors are related to the onset of cancer but not to the success of recovery from it.
C
Psychological factors are not related to the onset of cancer but are related to the success of recovery from it.
D
Psychological factors are related to neither the onset of cancer nor the success of recovery from it.

A

The Correct Answer is “C”
A number of theories regarding the relationship between psychological factors and the onset of cancer have been proposed. For instance, some authors have proposed that the “Type C” personality, typically described as a cooperative, unassertive patient who suppresses anger and complies with external authorities, is at a higher risk for cancer. However, most research shows that psychological factors and stressful events have a small or no effect on cancer incidence. By contrast, psychological factors do appear to be related to recovery from cancer. For instance, psychological treatments combining support and training in self-hypnosis are associated with higher survival rates and improved quality of life in cancer patients.

How well did you know this?
1
Not at all
2
3
4
5
Perfectly
59
Q
The type of racism exemplified by the lack of quality health care, delivery of substandard health care services and unavailability of preventive health services in minority communities is:
A
institutional racism
B
personally mediated racism
C
internalized racism
D
subtle racism
A

The Correct Answer is “A”
A. Institutional racism refers to restriction or denial of material conditions and access to power to members of minority groups. Personally mediated racism (b.) refers to prejudice and discrimination at the individual level. Internalized racism (c.) refers to acceptance of negative messages about the abilities and intrinsic worth by members of the stigmatized races. Subtle racism (d.) refers to the beliefs, attitudes, and actions of individuals (rather than institutions) and is a less obvious form of racism, which has been proposed to have replaced overt or “old-fashioned” prejudice and discrimination.

How well did you know this?
1
Not at all
2
3
4
5
Perfectly
60
Q

Use of benzodiazepines is associated with a “rebound effect.” If an individual is experiencing “rebound
anxiety,” this refers to:
A
an initial paradoxical increase in anxiety
B
anxiety persisting even after increasing dosage
C
anxiety re-appearing after long-term usage of the drug
D
an increase in severity of anxiety temporarily following the discontinuation of the drug

A

The Correct Answer is “D”
D. The “rebound effect” occurs when a benzodiazepine is discontinued and wears off. The symptoms come back stronger and the initial symptoms are magnified. When the medication is stopped, individuals can experience withdrawal symptoms as well as experience a “rebound” or resurgence of the feelings or condition the drug was originally managing. Essentially, the body/brain reacts to the drug by trying to overcome its effect, and once the drug is stopped that overreaction continues for awhile until it “gets through” to the body/brain that there is no longer a chemical to fight against. The symptoms lessen as the body readjusts to life without the chemicals. This “rebound effect” is also common for other conditions and drugs, such as rebound sinus congestion after frequent use of nasally inhaled decongestants or rebound headaches after a long-term course of painkillers for someone suffering from chronic headaches.

How well did you know this?
1
Not at all
2
3
4
5
Perfectly
61
Q
The best predictor of treatment outcome among adult substance abusers is:
A
age
B
ethnicity
C
history of criminal behavior
D
severity of substance abuse problems
A

The Correct Answer is “D”
Most studies have found that the best predictors of treatment outcome for substance abusers are psychiatric severity and problem severity. Other predictive factors include motivation and coping skills at baseline (J. R. McKay & R. V. Weiss, A review of temporal effects and outcome predictors in substance abuse treatment studies with long-term follow-ups: Preliminary results and methodological issues. Evaluation Review, 2001, 25(2), 113-161).

How well did you know this?
1
Not at all
2
3
4
5
Perfectly
62
Q
The effects of parental discipline on the development of conscience in toddlers is mediated by the toddlers' level of:
A
fearfulness
B
intelligence
C
activity level
D
pain threshold
A

The Correct Answer is “A”
Research by Kochanska (1997) has indicated that toddlers’ level of fearfulness mediates the effects of parental discipline. Specifically, the use of “gentle discipline” was found more effective for the development of conscience among fearful toddlers than among fearless toddlers. Fearless toddlers, on the other hand, developed conscience better through the use of a secure mother-child attachment. It should be noted that these results were primarily found during the toddler years and were not significant or were less significant when reassessed during preschool years [Multiple pathways to conscience for children with different temperaments: From toddlerhood to age 5, Developmental Psychology, 33(2), 228-240].

How well did you know this?
1
Not at all
2
3
4
5
Perfectly
63
Q

Anger management training for children has been criticized for:
A
limited positive effects due to focusing on the individual
B
limited positive effects due to focusing on the behavior
C
being developmentally inappropriate due to children’s tendency to blame anger on others
D
being developmentally inappropriate due to children’s inability to control feelings and behaviors

A

The Correct Answer is “A”
A. One criticism of anger management training for children has been that most programs, which utilize a cognitive-behavioral approach, place the emphasis on the individual’s perceptions, feelings and behaviors while ignoring the interpersonal or systemic factors. Research indicates that training program effectiveness is increased by expanding beyond the individual to include family, peer and community relationships. (See: Morley, E. & Rossman, S.B. (1997). Helping At-Risk Youth: Lessons From Community-Based Initiatives, Washington D.C., The Urban Institute.)

How well did you know this?
1
Not at all
2
3
4
5
Perfectly
64
Q
Latane called this a social disease. It occurred when high-level employees were assessed on a combined effort. It turned out they produced less than when they were working individually. It's called:
A
Social loafing
B
Group think
C
Social polarization
D
Social facilitation
A

The Correct Answer is “A”
Social loafing or Latane’s “social disease” is the discovery that in regard to work, individual output declines when people are working together as a group. However, social loafing does not occur under all conditions. It is reduced or eliminated when participants believe that their individual contributions are identifiable or uniquely necessary for the group to succeed. (B) “groupthink” is the tendency for a group to make an irrational or impulsive decision in order to reach consensus. (C) “Group polarization” refers to the tendency of individuals who start with a similar view to end with a more extreme position after group discussion.

How well did you know this?
1
Not at all
2
3
4
5
Perfectly
65
Q

Bilingual children ______________________ than monolingual children.
A
perform no better or no worse academically
B
perform worse academically
C
have no better or no worse intellectual flexibility
D
have increased intellectual flexibility

A

The Correct Answer is “D”
Evidence suggests that bilingual children perform better on tests of cognitive flexibility, divergent thinking, and metalinguistic awareness, than monolingual children. Findings on the academic performance of bilingual children are inconsistent and influenced by many factors such as the age of second language acquisition, community support, and academic services.

How well did you know this?
1
Not at all
2
3
4
5
Perfectly
66
Q
Cognitive dissonance theory suggests that which of the following clients would be most likely to report benefitting from psychotherapy?
A
a wealthy client who pays a very high fee
B
a poor client who pays a very high fee
C
a wealthy client who pays a very low fee
D
a poor client who pays a very low fee
A

The Correct Answer is “B”
According to the theory of cognitive dissonance, a person is motivated to reduce the negative, aversive state that results when his or her cognitions conflict with each other. From the perspective of this theory, the poor client paying a very high fee would experience a state of dissonance. Therefore, this client, more than those in the other responses, would be motivated to believe that he or she is benefitting from therapy.

How well did you know this?
1
Not at all
2
3
4
5
Perfectly
67
Q
In comparison studies of younger and older adults, it has been found that depression in older adults is least likely to result in
A
difficulties with memory problems.
B
anxiety feelings.
C
feelings of hopelessness.
D
expressed sadness.
A

The Correct Answer is “D”
Older adults are less likely than younger adults to express feelings of depression or sadness. They are more willing to express feelings of hopelessness (answer C) and anxiety (answer B). They are also more apt to have memory problems (answer A).

How well did you know this?
1
Not at all
2
3
4
5
Perfectly
68
Q
The incidence of Obsessive-Compulsive Disorder begins to differ for males and females at which age:
A
3 years
B
6 years
C
12 years
D
18 years
A

The Correct Answer is “B”
Obsessive-Compulsive Disorder has an earlier peak onset for males than females. For males the peak onset is between ages 6 and 15, and for females it is between ages 20 and 29. Thus, gender differences for OCD begin to become apparent at 6 years. However, in adulthood the incidence is about the same for both genders.

How well did you know this?
1
Not at all
2
3
4
5
Perfectly
69
Q

A psychologist is planning to retire in the next few months and has been decreasing her caseload. One of the psychologist’s former clients, who had terminated a few months earlier, phones the psychologist and reports having suicidal thoughts. The psychologist should:
A
inform the client that she is retiring and that she cannot accept any clients
B
refer the client to a colleague immediately
C
meet with the client until the client no longer needs treatment
D
meet with the client until the situation is stabilized and then refer

A

The Correct Answer is “D”
Although this question is referring to a former client, due to the potential danger for a suicidal client, the most appropriate action would be to meet with the client until the situation is stabilized and then refer.

How well did you know this?
1
Not at all
2
3
4
5
Perfectly
70
Q

The most consistent predictors of adolescent suicide are:
A
Depression, use of drugs and alcohol, antisocial behavior
B
Depression, loss of popularity, avoidance of social situations
C
Use of drugs and alcohol, body weight, level of social acceptance
D
Use of drugs, cigarette smoking, depression

A

The Correct Answer is “A”
Research on risk factors is not entirely consistent. However, most studies concur with Gould (1992) who found the most consistent predictors to be a diagnosis of depression, use of drugs and alcohol, and antisocial behavior.

How well did you know this?
1
Not at all
2
3
4
5
Perfectly
71
Q
Which of the following is not a characteristic of Western culture-bound values?
A
cause-effect approach
B
openness and intimacy
C
clear distinction between mental and physical well-being
D
strict adherence to a schedule
A

The Correct Answer is “D”
D. Sue and Sue (1999) described three premises from the Western perspectives of counseling, based on the assumption of individualism, that can have an effect on the therapeutic relationship. They are identified as class-bound values, culture-bound values and language variables. Class-bound values include valuing of time boundaries or a strict adherence to time schedules, an ambiguous and unstructured approach to problem solving, and the emphasis on long- range goals and solutions. Culture-bound values focus on individualism versus collectivism, cause and effect relationships for client problems (a.), emphasis on emotional/verbal expressiveness, active participation and openness to discussing intimate aspects of the client’s life (b.), and the separation of physical and mental well-being (c.). Language variables are those in which standard English and verbal communication are stressed.

How well did you know this?
1
Not at all
2
3
4
5
Perfectly
72
Q

Mothers who get high scores on the WAIS-III tend to have children who get high scores on the WISC-III. On the basis of this information, which of the following conclusions is most justified?
A
intelligence is hereditary
B
parental intelligence is correlated with offspring intelligence
C
the WAIS-III and the WISC-III are correlated
D
the WAIS-III and the WISC-III are uncorrelated

A

The Correct Answer is “B”
None of these answers is great, but the only one that is possible is that parental intelligence and offspring intelligence are correlated. To know this for sure, we would need to know more about moderate and low scorers on the WAIS-III – do their children have moderate and low scores, respectively on the WISC-III? However, none of the other choices makes any sense. For instance, we can’t say that intelligence is hereditary from this information, since environmental rather than genetic factors may have resulted in the similarity of scores between mother and child. Also, we can’t say the WISC-III is correlated (or uncorrelated) with the WAIS-III. To measure the correlation between two tests, one must administer both of them to the same set of examinees.

How well did you know this?
1
Not at all
2
3
4
5
Perfectly
73
Q

Which of the following statements regarding therapist-client sexual relations is, according to research, most correct?
A
Therapists treating clients with a diagnosis of Borderline Personality Disorder, Histrionic Personality Disorder, or who have a history of incest are more likely to become sexually involved with their patients than other therapists.
B
Therapists who become sexually involved with their patients are likely to have less formal education or to have received less professional recognition than other therapists.
C
Therapists who have completed a personal course of therapy are less likely to become sexually involved with their patients than therapists who have not undergone therapy.
D
Therapists who become sexually involved with a patient typically do so with a patient who is significantly younger.

A

The Correct Answer is “D”
Kenneth Pope, in an appendix of his book Sexual Feelings in Psychotherapy (Washington, DC: APA, 1993), provides a review of the research in the area of client-therapist sexual relations. Only choice D reflects research findings – one study found that the average ages of therapists and clients sexually involved with each other were 43 and 33, respectively; another study found these ages to be 42 and 30. In contrast to choice A, no research supports the notion that a particular diagnosis or history places a patient at greater risk for sexual involvement with a therapist. Choice B is also incorrect – in fact, one study found that psychologists who had attained a high level of professional achievement reported a higher rate of sex with patients. Similarly, in regard to choice C, one study found that psychologists who had completed psychotherapy or psychoanalysis were more likely to have become sexually involved with a patient than those therapists who hadn’t undergone therapy. According to Pope, the best single predictor of sexual exploitation in therapy is a therapist who has exploited a patient in the past.

How well did you know this?
1
Not at all
2
3
4
5
Perfectly
74
Q
Which of the following medications would be least likely to cause impaired concentration as a side effect?
A
clomipramine
B
tofranil
C
anafranil
D
fluoxetine
A

The Correct Answer is “D”
Choices “A”, “B”, and “C” are all tricyclic anti-depressants. In fact, choices “A” and “C” are different names for the same medication; Anafranil is the brand name for clomipramine. Choice “D”, fluoxetine (brand name: Prozac) is one of the selective serotonin reuptake inhibitors (SSRIs). SSRIs have fewer and less severe side effects than tricyclics, and they are generally not associated with cognitive impairments. By contrast, tricyclics are associated with cognitive impairments such as deficits in concentration – especially in the elderly

How well did you know this?
1
Not at all
2
3
4
5
Perfectly
75
Q
It has been found that abused children often cling to their abusive parents. This can be explained in behavioral terms as the effect of
A
extinction.
B
delayed conditioning.
C
intermittent reinforcement.
D
spontaneous recovery.
A

The Correct Answer is “C”
Most abusing parents are abusive only some of the time. Other times they are quite loving and protective. In behavioral terms, this means that the child is on an intermittent reinforcement schedule, and, more specifically, a variable ratio schedule. In other words, any given behavior, including clinging, will be reinforced after a variable and unpredictable number of responses. Intermittent reinforcement schedules, and especially variable ratio schedules, tend to produce behaviors that are difficult to extinguish. This could explain why the clinging behavior continues.

How well did you know this?
1
Not at all
2
3
4
5
Perfectly
76
Q

A kappa coefficient of .93 would indicate that the two tests
A
measure what they are supposed to.
B
have a high degree of agreement between their raters.
C
aren’t especially reliable.
D
present test items with a high level of difficulty.

A

The Correct Answer is “B”
The kappa coefficient is used to evaluate inter-rater reliability. A coefficient in the lower .90s indicates high reliability. Answer A is a layman’s definition of the general concept of valdity.

How well did you know this?
1
Not at all
2
3
4
5
Perfectly
77
Q

The difference between the multiple hurdle and the multiple cut-off selection technique is that, in multiple hurdle,
A
examinees must succeed on all predictors.
B
not all predictors are administered to all examinees.
C
predictors are specially designed to be “culture-fair.”
D
predictive validity is empirically established.

A

The Correct Answer is “B”
Both multiple hurdle and multiple cut-off involve administering a series of predictor measures to prospective applicants. And in both cases, the examinee must succeed on all predictors in order to be selected. The difference is that, in multiple hurdle, predictors are administered in a successive order. If the applicant fails on any predictor, he or she no longer completes subsequent ones. By contrast, when multiple cut-off is used, examinees usually take all the predictors and the predictors are not necessarily administered in any particular order.

How well did you know this?
1
Not at all
2
3
4
5
Perfectly
78
Q

The difference between insanity and psychosis is that:
A
insanity is a legal term and psychosis is a term used in mental health literature
B
insanity is an outdated term and is no longer applied in legal cases, while psychosis is used both in mental health and legal literature
C
insanity is legally a broader term, and subsumes the term psychosis
D
insanity refers to disturbances in thought and emotion, while a psychotic disturbance, by definition, affects thought only

A

The Correct Answer is “A”
Insanity is strictly a legal term; it usually means that a person is in such a mental state that he or she cannot distinguish between right or wrong. Psychosis, on the other hand, is a psychiatric term.

How well did you know this?
1
Not at all
2
3
4
5
Perfectly
79
Q
The Wisconsin Card Sorting Test is useful for the detection of damage to the:
A
frontal lobe
B
temporal lobe
C
hippocampus
D
corpus callosum
A

The Correct Answer is “A”
The Wisconsin Card Sorting Test (WCST) is used primarily to assess perseveration and abstract reasoning. However, it has also been found to be a useful measure of executive (frontal lobe) functioning and in discriminating between frontal and non-frontal lesions. Subjects must sort 64 cards using different strategies, for example, by color, form (triangle, star, star, and circle) or number of items on the cards.

How well did you know this?
1
Not at all
2
3
4
5
Perfectly
80
Q

A child has been assigned a new legal guardian. His biological parents had a psychological evaluation completed and had received the results. The new legal guardian is requesting a copy of the report. What is the most correct thing to do?
A
Get a signed release from the biological parents
B
Provide the legal guardian with a copy of the report
C
Send the report to the court mediator
D
Keep the evaluation on file until the child is emancipated; then forward to him

A

The Correct Answer is “B”
As legal guardian he or she has the right to a copy of the report. The key term is “legal”. A court has determined legal access in this case so you as the psychologist only need to be guided by the law.

81
Q

Introducing a goal that requires cooperation between conflicting intergroups results in:
A
increased hositility, increased competition
B
increased hostility, reduced competition
C
reduced hositility, increased competition
D
reduced hostility, reduced competition

A

The Correct Answer is “D”
The introduction of a superordinate goal, one that requires cooperation to accomplish, was found to be the most effective way of reducing or alleviating intergroup hostility and competition in Sherif et al.’s “Robber’s Cave” study.

82
Q
Which of the following correlation coefficients is used to assess convergent validity:
A
heterotrait-monomethod
B
monotrait-heteromethod
C
heterotrait-heteromethod
D
monotrait-monomethod
A

The Correct Answer is “B”
The response choices make up a multitrait-multimethod matrix, a complicated method for assessing convergent and discriminant validity. Convergent validity requires that different ways of measuring the same trait yield the same result. Monotrait-heteromethod coefficients are correlations between two measures that assess the same trait using different methods; therefore if a test has convergent validity, this correlation should be high. Heterotrait-monomethod and heterotrait-heteromethod both confirm discriminatory validity, and monotrait-monomethod coefficients are reliability coefficients.

83
Q

According to both Kohlberg’s and Piaget’s theory of moral development, which of the following statements is not true?
A
Moral development is rooted in cognitive development.
B
Appropriate peer interaction promotes moral development.
C
Moral development occurs in an invariant sequence of stages.
D
There may be differences across cultures in the development of moral reasoning, especially in the early stages.

A

The Correct Answer is “D”
Although Kohlberg is probably better known as a theorist of moral development than Piaget, the latter had his own theory, which basically stressed that as children grow, they move from viewing rules as fixed dictates of authority to flexible instruments of human purposes that can be changed. The point of view of Kohlberg, Piaget, and other theorists with similar ideas is sometimes termed the cognitive-developmental point of view because, from the perspective of these theories, development in a number of areas is mainly a function of cognitive development. Both Kohlberg and Piaget believed that moral development occurs in an invariant sequence and is related to both cognitive development and peer interaction. However, neither Piaget nor Kohlberg believed that there are cultural differences in the early stages of moral development, though Kohlberg has noted that the nature and rate of moral development at its higher levels does differ across cultures.

84
Q

What did Vygotsky stress over Piaget?
A
developmental levels of specific children
B
Cognitive development of children
C
Their relationship with others in the family
D
The importance of the accommodation stage

A

The Correct Answer is “C”
Vygotsky stressed the importance of social and cultural impact on the developing child much more than Piaget did. These factors included the family and the child’s milieu.

85
Q
The stages of the Minority Identity Development (MID) model developed by Atkinson, Morten, and Sue (1994) reflect changes in which of the following?
A
acculturation
B
ego development
C
locus of control
D
self-awareness
A

The Correct Answer is “A”
You may have had trouble with this one, but if you’re familiar with the MID model and the characteristics of its stages (conformity, dissonance, resistance and immersion, introspection, synergetic articulation), you may have recognized that each stage involves a different combination of attitudes toward one’s own culture and the majority culture and, therefore, reflects different levels of acculturation.

86
Q
In cases of sexual abuse, the abuser is typically:
A
A relative or close family friend
B
A stranger
C
A same-age sibling
D
An alcoholic
A

The Correct Answer is “A”
D. Finkelhor (Sexually Victimized Children New York: Free Press 1979) reports that the abuser is typically
either a relative or close family friend of the victim. In assessing for sexual abuse, clinicians take into account “power differentials” and thus in the case of a same-age sibling, (Answer C), it may be considered sexual exploration or play.

87
Q
When a person with deep dyslexia is presented with the written word "cat," he is most likely to respond with which of the following?
A
"tac"
B
"dog"
C
"at"
D
"I don't know"
A

The Correct Answer is “B”
People with deep dyslexia exhibit a number of reading errors including semantic paralexia, which involves producing a response that is similar in meaning to the target word (e.g., dog for cat or arm for leg).

88
Q
Which of the following would be used to measure the internal consistency of a test?
A
kappa coefficient
B
test-retest reliability
C
split-half reliability
D
alternate forms reliability
A

The Correct Answer is “C”
Internal consistency is one of several types of reliability. As its name implies, it is concerned with the consistency within a test, that is, the correlations among the different test items. Split-half reliability is one of the measures of internal consistency and involves splitting a test in two and correlating the two halves with each other. Other measures of internal (inter-item) consistency are the Kuder-Richardson Formula 20 (for dichotomously scored items) and Cronbach’s coefficient alpha (for multiple-scored items). Test-retest reliability (“B”) is not concerned with internal consistency, but rather, the stability of a test over time, and uses the correlations of scores between different administrations of the same test. Alternative forms reliability (“D”) is concerned with the equivalence of different versions of a test. And the kappa coefficient is used as a measure of inter-rater reliability, that is, the amount of agreement between two raters.

89
Q

Which of the following is the clearest indication that a person is suffering from a substance addiction?
A
The person denies that he or she has a problem.
B
The person displays clear signs of tolerance and/or withdrawal.
C
The person uses the substance in situations when use is physically hazardous, such as when driving.
D
The person sometimes misses work because he or she is intoxicated or hung over.

A

The Correct Answer is “B”
Of the choices listed, marked tolerance and withdrawal are the most telling signs that a person should receive a diagnosis of Substance Dependence. Tolerance and withdrawal are among the diagnostic criteria for Substance Dependence. As for the other choices, C and D sound like the criteria for Substance Abuse, which according to DSM-IV, are that the person displays one or more of the following within a 12 month period: 1) recurrent substance use resulting in a failure to fulfill major school, work, or home obligations; 2) recurrent use in situations in which use is physically hazardous; 3) recurrent substance-related legal problems; and 4) continued substance use even though it causes or exacerbates a social or interpersonal problem. Although these signs may be present in Substance Dependence, they are not as clear a sign of Dependence as tolerance and withdrawal. Denial of the problem is not a diagnostic criterion for Dependence. Clearly, although individuals who are dependent are often in denial, not all those who deny they have a problem actually do have one.

90
Q

To testify as an “expert witness” in a court case, you must
A
obtain certification as a forensic psychologist.
B
obtain certification in the specific area you are testifying about.
C
be recognized by both sides’ attorneys as an expert.
D
have specialized knowledge in the area about which you are testifying.

A

The Correct Answer is “D”
There are no special certifications that psychologists must obtain in order to testify as expert witness. Nor do they need to be recognized by both parties in the legal proceeding as an expert – in fact, in a substantial proportion of cases, the psychologist’s expertise is likely to be challenged by one of the parties. From an ethical standpoint, the relevant requirement is that psychologists testifying as experts (like all other psychologists) should not practice outside the scope of their competence. In other words, psychologists representing themselves as experts in a particular area should have specialized knowledge in that area.

91
Q
The authoritative style of parenting typically results in better school performance among:
A
Asians
B
Hispanics
C
African-Americans
D
Whites
A

The Correct Answer is “D”
Overall, the authoritative style of parenting (as compared to the authoritarian, permissive, and uninvolved styles) has the most positive outcomes in children’s growth and development – including academic achievement. However, researchers have found that the benefits of authoritative parenting are moderated by ethnicity. Specifically, Hispanics and African-Americans do not appear to benefit as much as Whites from having authoritative parenting. And, although Asian-Americans tend to do well academically, they are least likely to have authoritative parents.

92
Q

From a legal standpoint, a charge of malpractice against a psychologist would be held valid if it were proven that the therapist:
A
had a duty of care to the patient, held malevolent intentions toward the patient, and engaged in activity that resulted in harm to the patient.
B
had a duty of care to the patient and held malevolent intentions toward the patient; demonstration of actual harm is not necessary.
C
had a duty of care to the patient and engaged in activity that resulted in harm to the patient; malevolent intentions need not be present.
D
engages in activity that harms a patient; malevolent intentions and a duty of care are not necessary.

A

The Correct Answer is “C”
For a malpractice claim against a psychologist to be held valid, three elements must be proven: 1) the psychologist must have had a professional relationship with (and, therefore a duty of care to) the client; 2) the psychologist must have been negligent or failed to live up to that duty; and 3) harm to the patient must have resulted. In other words, the psychologist’s intentions are not an issue

93
Q

A friend complains to you that his four-year old frequently lies to him even when the child will not be punished or experience other negative consequences for not telling the truth. Based on your familiarity with the literature on deception in children, you
A
tell your friend that the child can’t be lying because children are not cognitively capable of doing so until age six or seven.
B
tell your friend that the child must be anticipating negative consequences because, at age four, children lie only in order to avoid punishment.
C
tell your friend that, without any other symptoms, the child’s behavior is probably normal since young children lie for several reasons besides avoiding punishment.
D
tell your friend that the child should be evaluated since lying at such an early age is often a warning sign of pathology.

A

The Correct Answer is “C”
Although Piaget and others have argued that young children are cognitively incapable of lying, the research has found that children as young as three lie for various reasons (e.g., to avoid punishment or embarrassment, to obtain rewards).

94
Q
An African-American family presents for family therapy. From a systems perspective, it would be most advisable to:
A
focus on multi-generational issues.
B
see each family member individually.
C
use behavioral techniques.
D
educate family members about community resources
A

The Correct Answer is “A”
You probably could answer this one correctly by focusing on the word “systems” since, of the choices, focusing on more than one generation most implies that you would be looking at the entire family system. In addition, African-American culture, as compared to Anglo culture, tends to be characterized by a greater emphasis placed on relationships with extended family members.

95
Q

A student who is aware of the serial position effect, should pay more attention to:
A
material studied at the beginning of a study session
B
material studied at the end of a study session
C
material studied during the middle of a study session
D
an older sibling’s advice

A

The Correct Answer is “C”
The serial position effect refers to the tendency to best recall the first (primacy effect) and last (recency effect) items studied. Thus, a student who is aware of this phenomenon should pay more attention to the material studied during the middle of a study session, since that material is most likely to be forgotten.

96
Q
The tendency to experience losses more heavily than gains of equal magnitude results in less risky decisions and is referred to as:
A
risky shift
B
adverse impact
C
satisficing
D
loss aversion
A

The Correct Answer is “D”
Loss aversion is one component in Kahneman and Tversky’s prospect theory. Loss aversion refers to the tendency to base decisions more heavily on the fear of loss than the hope of gain. For example, if you heard from a friend that his mechanic did a good job fixing his car but another friend told you that the same mechanic did a poor job fixing his car, you would be less likely to risk using that mechanic. The other answer choices are terms that you should be familiar with. Risky shift (A) occurs when a group makes a decision that is riskier than would have been made by the members individually. Adverse impact (B) occurs when the hiring rate for a minority group is less than 80% of the hiring rate of the majority group. Satisficing (C) refers to the decision-making style of using the minimal amount of information to reach a “good enough” solution.

97
Q
According to research on sex-role determinants, the characteristic that is most related to genetics as opposed to socialization is:
A
sociability.
B
dependency.
C
aggression.
D
anxiety.
A

The Correct Answer is “C”
Almost all empirical studies show that very few abilities that distinguish males from females are biologically determined. That is, developmental psychologists tell us that the things we see typically as feminine and things we see typically as masculine are defined and developed through socialization. There are however a couple of characteristics that seem to transcend socialization. From the four possibilities listed here, aggression is the one that we feel is more related to hormones than to learning. Boys are just more active and have higher aggressive drives than girls do.

98
Q
You are commuting to work early in the morning at 15 mph over the speed limit. You glance in your rearview mirror and notice blue flashing lights. You slow down and pull over. According to French and Raven's ideas about social power, which type of power are you attributing to the policemen?
A
legitimate
B
reward
C
informational
D
referent
A

The Correct Answer is “A”
According to French and Raven (1959), in the case of legitimate authority, the target believes the influencing agent has legitimate authority. With reward power (response B), the influencing agent has control over valued rewards and resources. And, what you are about to receive from the policeman will not be a reward. With informational power (response C), the influencing agent possesses specific information needed by the target person, and with referent power (response D), the target identifies with or likes the influencing agent.

99
Q
Repetitive exercise and hypnosis are therapeutic techniques used to establish the “relaxation response” which involves \_\_\_\_\_\_\_\_\_\_\_\_\_\_\_\_\_\_\_\_\_\_\_\_\_\_\_\_ activity.
A
decreased parasympathetic nervous system
B
decreased sympathetic nervous system
C
increased somatic nervous system
D
increased autonomic nervous system
A

The Correct Answer is “B”
B. The relaxation response is, in effect, the opposite of the “fight or flight” response to stressful or threatening situations which over time may produce hypertension, cardiac and other problems that may seriously affect health. Herbert Benson and his colleagues (1971) studied the affects of meditation on people with high blood pressure brought on by the everyday stress of living and described a physiological response that decreased sympathetic nervous system activity resulting in decreased metabolism, decreased heart rate, decreased blood pressure, and decreased rate of breathing, as well as slower brain waves. This reaction was coined the “relaxation response.” Further findings indicate relaxing just 20 minutes each day can be beneficial to both physical and mental health.

100
Q

Dementia due to Head Trauma
A
is usually progressive in cases of moderate to severe trauma.
B
is usually progressive only in cases of repeated head trauma.
C
unlike other forms of Dementia, does not usually involve disturbances in executive functions.
D
unlike other forms of Dementia, is associated more with deficits in executive functions than with memory impairment.

A

The Correct Answer is “B”
As described in DSM-IV, Dementia due to Head Trauma is usually nonprogressive when it is the result of a single head trauma. If it is progressive, this usually indicates a superimposed condition (e.g., hydrocephalus).

101
Q
A young man is in a car accident and apparently receives some brain injury. Two days later, the man has a 10 a.m. appointment with a neurologist to evaluate the extent of his injury. Later that same day, the young man encounters the neurologist in a coffee shop but doesn't recall the neurologist, their meeting, or the evaluation. The young man is apparently suffering from
A
paraprosopia.
B
prosopagnosia.
C
anterograde amnesia.
D
retrograde amnesia.
A

The Correct Answer is “C”
In this situation, the patient seems to be having trouble forming new memories. This is referred to as anterograde amnesia. Paraprosopia (answer A) involves visual hallucinations of terrifying faces; prosopagnosia (answer B) is the inability to recognize familiar faces.

102
Q

The director at the training agency, where you are a supervisor, has promised two new interns that they will have plenty of testing experience as part of their training. In order to meet quota, you are under a great deal of pressure to bill many hours. One intern has a great deal of previous testing experience and the other intern has very little. The majority of the testing cases are given to the more experienced intern, and as a result this intern is given the majority of your time and attention. This situation is:
A
ethical
B
unethical
C
unethical because you are exploiting the more experienced intern
D
unethical because the less experienced intern is not getting the training as it was described to him by the director of the clinic

A

The Correct Answer is “A”
There is no indication that the more experienced intern is being exploited (response “C”) and while the interns are not receiving an equal number of testing opportunities, the situation is not unethical solely because the less experienced intern is getting fewer testing cases and/or attention. The director stated the interns would have “plenty of testing experience,” not necessarily an equal amount of testing cases, time, or attention. Additionally, the definition of plenty is relative to the individual intern, their previous education, training, and experience. Another relative term in the question to consider is “majority.” It would be unethical to have the intern with very little testing experience perform more than his/her current level of competency (Standard 2.05(2): Delegation of Work to Others). Note: this intern is receiving test cases, time, and attention – just not the “majority” of cases and attention.

103
Q
A post-menopausal woman is most likely to find that her reduced level of estrogen has
A
decreased her ability to become sexually aroused.
B
reduced her overall interest in sex.
C
made intercourse uncomfortable.
D
made it easier to achieve orgasm.
A

The Correct Answer is “C”
Because of its effects on vaginal tissues, a lower estrogen level can cause discomfort during intercourse. However, it is not associated with changes in other aspects of sexual functioning including interest, arousal, or drive.

104
Q
Korsakoff's syndrome is a form of Alcohol-Induced Persisting Amnestic Disorder that is due to
A
dehydration.
B
thiamine deficiency.
C
vitamin C deficiency.
D
sensory impairment.
A

The Correct Answer is “B”
The DSM-IV refers to Korsakoff’s syndrome as Alcohol-Induced Persisting Amnestic Disorder due to Thiamine Deficiency. The disorder is characterized by anterograde and often retrograde amnesia, and causes damage to neurons in the thalamic and hypothalamic brain regions. When a person displays early signs of the disorder, it can sometimes be prevented through large doses of thiamine. However, once brain neurons become damaged, thiamine treatment cannot reverse the syndrome.

105
Q

Brief dynamic therapy is least likely to involve
A
a focus on specific problems.
B
an emphasis on restoring the person to a previous level of normal functioning.
C
an attempt to engender insight into the unconscious.
D
the use of techniques such as free association and dream interpretation.

A

The Correct Answer is “B”
B sounds more like the goal of crisis intervention than that of brief psychotherapy. Brief dynamic therapy is focused on specific symptoms; whereas, the goal of long-term psychodynamic therapy is usually global personality change. These symptoms are usually viewed as pathological; in other words, the emphasis is on altering the normal level of functioning rather than returning the person to it. C and D both describe possible aspects of brief dynamic therapy. Techniques of long-term dynamic therapy, such as free association and dream interpretation, may be used. In addition, there may be an attempt to engender insight into the unconscious, but (unlike as in long-term therapy) the patient is left to assimilate this insight on his or her own.

106
Q
What is needed for a successful malpractice lawsuit against a psychologist?
A
A stated professional relationship
B
Significant neglect
C
Severe damage
D
Sufficient harm
A

The Correct Answer is “D”
For a successful malpractice suit, according to Behnke, Preis and Bates in their provocative book, California Mental Health Law WW Norton and Company 1998, one must have the four “D’s” to win a malpractice suit: “dereliction of a duty directly causing damage.” One must show that there was a duty–-(which assumes a professional relationship, although it does not need to be explicitly stated as in choice “A”), the duty was not carried out; and one must show, that as a direct result of this, the person suffered damages.

107
Q
A predictor that is highly sensitive for identifying the presence of a disorder would most likely result in:
A
measurement error
B
type II error
C
a high number of false positives
D
a high number of false negatives
A

The Correct Answer is “C”
A predictor that is highly sensitive will more likely identify the presence of a characteristic; that is, it will result in more positives (true and false). This may be desirable when the risk of not detecting a problem is high. For example, in the detection of cancer, a blood test that results in a high number of false positives is preferable to one that has many false negatives. A positive test result can then be verified by another method, for example, a biopsy. Measurement error (A) is the part of test scores which is due to random factors. Type II error (B) is an error made when an experimenter erroneously accepts the null hypothesis.

108
Q

When scoring the Rorschach, “form quality” is a measure of the degree to which:
A
determinants are fully integrated
B
a response meaningfully integrates two or more adjacent detail areas
C
the examinee’s response corresponds to the actual structure of the inkblot
D
the examinee’s response makes use of the entire inkblot

A

The Correct Answer is “C”

Form quality refers to the degree to which an examinee’s response is consistent with the form (shape) of the inkblot

109
Q
manager assumes that his employees need constant supervision and detailed guidelines in order to perform their jobs well. What management style does this reflect?
A
Theory Y
B
Theory X
C
laissez-faire
D
management by objectives
A

The Correct Answer is “B”
McGregor’s (1960) Theory X and Theory Y management theories reflect the key differences between the scientific management and human relations administrative models. Managers adopting a scientific management model assume that workers are motivated primarily by financial self-interest (i.e., pay is the most effective motivator); that job demands must match workers’ skills; and that workers need detailed guidelines and constant supervision because they are incapable of regulating themselves or assuming responsibility on their own. Theory X is similar to scientific management: Theory X managers believe that workers dislike work and avoid it whenever possible; therefore, workers must be directed and controlled.

110
Q
Following a stroke, a woman complains of numbness in her left hand and she does not respond to images presented to her left visual field. She most likely suffered damage in:
A
Broca's area
B
Wernicke's area
C
the right precentral gyrus
D
the right postcentral gyrus
A

The Correct Answer is “D”
The postcentral gyrus includes the somatosensory cortex, and is located in the parietal lobe. Damage to the right postcentral gyrus could result in numbness in the left hand and contralateral neglect, which includes neglect of the left visual field. Broca’s (“A”) and Wernicke’s (“B”) areas are both located in the left hemisphere of the brain and only affect language. The precentral gyrus (“C”) is responsible for motor functioning.

111
Q
According to recent research, the compressed workweek has the least impact on:
A
absenteeism
B
supervisor performance ratings
C
overall job satisfaction
D
satisfaction with work schedule
A

The Correct Answer is “A”
A. A meta-analysis by Baltes and colleagues found that the compressed workweek (increasing the number of hours per day and decreasing the number of days worked in a given work period) did not have a strong impact on absenteeism or objective measures of job performance. However, compressed workweek schedules were found to have positive effects on supervisor ratings of employee performance, employee overall job satisfaction, and employee satisfaction with the work schedule [B. B. Baltes et al., (1999). Flexible and compressed workweek schedules: A meta-analysis of their effects on work, Journal of Applied Psychology, 84(4), 496-513]. Earlier conclusions about the effects of the four-day (compressed) workweek have not been completely consistent with the findings from this meta-analysis.

112
Q
A patient with diabetes and no prior history of a mental disorder recently had his leg amputated. For the past several weeks, he has had depressed mood and decreased interest in activities he used to enjoy. His diagnosis would most likely be:
A
Adjustment Disorder with Depressed Mood
B
Major Depressive Disorder, Single Episode
C
Acute Stress Disorder
D
Posttraumatic Stress Disorder
A

The Correct Answer is “A”
An Adjustment Disorder involves emotional or behavioral symptoms that develop in response to an identifiable stressor. The disturbance does not meet the criteria for another Axis I disorder. If the question indicated that the patient had additional depressive symptoms, a diagnosis of Major Depressive Disorder (B) might have been more appropriate. However, since only two symptoms were offered, Adjustment Disorder with Depressed Mood is the best option. For a diagnosis of Acute Stress Disorder (C) or Posttraumatic Stress Disorder (D), the person must be exposed to extreme stress followed by a specific set of symptoms including: dissociative symptoms, reexperiencing of the trauma, increased arousal, and avoidance.

113
Q
General symptoms that may accompany the third stage of Alzheimer's Dementia include
A
apathy and emotional blunting.
B
depression and anomia.
C
irritability and anger.
D
paranoia and labile mood.
A

The Correct Answer is “A”
These symptoms are characteristic of the third stage of Alzheimer’s Dementia according to many authors. The symptoms vary considerably from person to person and may include personality, behavioral, and cognitive changes. The symptoms of depression and anomia (answer B) and irritability and anger (answer C) are often seen during the first stage. Paranoia and labile mood (answer D) are characteristic of the second.

114
Q

According to Berry’s acculturation model, marginalization is characterized by:
A
low involvement in the mainstream culture.
B
low involvement in both the mainstream culture and the minority culture.
C
low involvement in the minority culture.
D
low involvement in the mainstream culture and high involvement in the minority culture.

A

The Correct Answer is “B”
Berry’s model of acculturation distinguishes between two dimensions – retention of the minority culture and maintenance of the majority culture. He proposes that these dimensions are independent, which means that a person can be high on one and low on the other; high on both; or low on both. Berry uses the term “marginalization” to describe the situation where a member of a minority group does not identify strongly with either culture.

115
Q

The Working Memory Index of the WAIS-III is based on which of the following subtests?
A
vocabulary, similarities, and information
B
digit symbol and symbol search
C
digit span, arithmetic, and letter-number sequencing
D
picture completion, block design, and matrix reasoning

A

The Correct Answer is “C”
The WAIS-III measures four factors: verbal comprehension, perceptual organization, working memory, and processing speed. The working memory index is comprised of digit span, arithmetic, and letter-number sequencing. Each of the other choices contain the subtests which comprise the other indices. Vocabulary, similarities, and information subtests (A) make up the verbal comprehension index. Digit symbol and symbol search (B) determine the processing speed index. And picture completion, block design, and matrix reasoning (D) comprise the perceptual organization index.

116
Q

The friend of a psychologist is the owner of a small publishing firm. The friend offers the psychologist substantial compensation if she would lend her name to endorsements for his publications, and at times advise and consult on book acquisitions and marketing plans. The psychologist:
A
could accept this as long as APA affiliation is not mentioned in the endorsements.
B
could accept this only if the publications she would be dealing with are in her area of competency.
C
could accept this only if the publications she would be dealing with are in her area of competency, and the endorsements will be based on the psychologist’s honest opinion of the publications.
D
should consult the local ethics committee before making her decision.

A

The Correct Answer is “C”
The ethical standards say that psychologists cannot make false, deceptive, fraudulent, or misleading statements. Unless the psychologist’s endorsements were based on sound expertise and her honest opinion of the publications in question, these standards would be violated.

117
Q
Which of the following descriptive words for tests are most opposite in nature?
A
speed and power
B
subjective and aptitude
C
norm-referenced and standardized
D
maximal and ipsative
A

The Correct Answer is “A”
Pure speed tests and pure power tests are opposite ends of a continuum. A speed test is one with a strict time limit and easy items that most or all examinees are expected to answer correctly. Speed tests measure examinees’ response speed. A power test is one with no or a generous time limit but with items ranging from easy to very difficult (usually ordered from least to most difficult). Power tests measure level of content mastered.

118
Q
Which is the most common anticholinergic side-effect?
A
Light tremor
B
Blurred vision
C
Diarrhea
D
Weight loss
A

The Correct Answer is “B”
B. Anticholinergic effects may be either central or peripheral. Central side effects are cerebral and include impaired concentration, confusion, attention deficit, and memory impairment. Peripheral side effects include dry mouth, constipation, urinary retention, bowel obstruction, dilated pupils, blurred vision, increased heart rate, and decreased sweating. Light tremor (a.) and diarrhea (c.) are not associated side-effects, and while dry mouth may affect appetite and result in weight loss (d.), it is not considered an anticholinergic side-effect either.

119
Q
Studies have been conducted to determine the relationship between number of therapy sessions and outcomes. Some of these studies demonstrate that, after only a few sessions, clients are most likely to show improvement in terms of which of the following?
A
insight
B
interpersonal skills
C
sense of hopelessness
D
stress
A

The Correct Answer is “C”
The primary benefit of a few sessions of therapy is its impact on feelings of despair and hopelessness, which many new therapy clients feel. Apparently, the reduced sense of hopelessness is due to the fact that the client begins to feel that therapy may actually be of some help.

120
Q

psychologist, who is a member of APA, receives a charge letter from the Ethics Committee. The charge against the psychologist is that he cheated on the psychology-licensing exam. This means that the psychologist
A
will be barred from further consideration for licensure.
B
will have to retake the exam under supervision.
C
must respond to the charge letter within thirty days.
D
can request independent adjudication or a formal hearing.

A

The Correct Answer is “C”
The Ethic Committee’s procedures are outlined in APA’s Rules and Procedures. When the Committee decides that it is appropriate to pursue a formal case investigation, it sends a charge letter to the complainee. The complainee then has thirty days to file an initial response. A request for independent adjudication or formal hearing (answer D) is not made until the Committee has actually found the psychologist to be guilty of an ethical violation.

121
Q
An effective behavioral treatment to reduce the symptoms of depression is Structured Learning Therapy (SLT). SLT contains all of the following except:
A
social skills training
B
overcorrection
C
modeling
D
role-playing
A

The Correct Answer is “B”
B. Structured Learning Therapy incorporates social skills training, an early approach to the treatment of depression, along with modeling (a.), role-playing (b.), skill instruction, and performance appraisal. (See: A. P. Goldstein, Psychological Skills Training: The Structured Learning Technique, New York, Pergamon Press, 1980.)

122
Q
Individuals with Tourette’s Disorder frequently suffer from a learning disorder in school. The most likely cause is:
A
expressive language disorder
B
attentional and hyperactivity problems
C
social problems
D
environmental stressors
A

The Correct Answer is “B”
B. Although learning problems are associated with the disorder, children with Tourette’s Syndrome (TS) as a group have the same range of IQ as the population at large. The etiology of learning disabilities (LD), as well as the most accurate conceptualization of them as either comorbid disorders or as prevalent, variable components of the broader TS phenotype, has yet to be determined. In a recent study of more than 3100 children with Tourette’s, ADHD was the most prevalent comorbid disorder occurring in 58% of subjects. Of those with TS plus learning disabilities, 80% also had a diagnosis of ADHD. The increased rates of ADHD in those diagnosed with TS + LD and the finding that only 11 % of the TS children without ADHD had a diagnosis of LD demonstrates the potential impact of ADHD on LD as a causal factor or as a confounder for the diagnosis of LD. (L. Burd, L., Freeman, R.D., Klug, M.G. and Kerbeshian, J. (2005). Tourette syndrome and learning disabilities, BioMed Central Pediatrics, 5)

123
Q
Early signs of Dementia due to HIV Disease are
A
cognitive slowing and apathy.
B
disorientation and aphasia.
C
impairments in executive functioning.
D
speech impairments and inappropriate social behavior.
A

The Correct Answer is “A”
Dementia due to HIV Disease includes cognitive, personality, and motor symptoms. Common symptoms include cognitive slowing, impaired attention, and forgetfulness; apathy and social withdrawal; and clumsiness and leg weakness.

124
Q

You are starting a stop-smoking group and tell a newspaper reporter who smokes that he can participate in the group for free if he’ll write an article about it for the local newspaper. This arrangement is
A
unethical.
B
ethical as long as you don’t tell the reporter what to write about.
C
ethical as long as you check to make sure that information about the group is accurate.
D
ethical as long as the confidentiality of the group participants is not violated.

A

The Correct Answer is “A”
This issue is addressed in Standard 5.02(b) of the Ethics Code, which states that “Psychologists do not compensate employees of press, radio, television, or other communication media in return for publicity in a news item.”

125
Q
If you are interested in determining whether the relationship between arousal and performance assumes a linear or a non-linear shape, the best statistical analysis to use would be
A
multiple regression analysis.
B
trend analysis.
C
logistic regression.
D
principal components analysis.
A

The Correct Answer is “B”
Trend analysis is a statistical technique used to determine the trend or shape that best describes the relationship between two variables. The technique basically involves collecting data on two variables and running statistical analyses to determine what trend or trends (e.g., linear, U-shaped) are significant. For example, in studying the relationship between arousal and performance, one could study 100 students and collect data on how aroused they are and how well they perform. Then, one could run a separate analysis for different types of trends and see which receives the strongest support.

126
Q

A meta-analysis differs from a literature review in that
A
a meta-analysis involves a review of research in a more broadly defined area than a literature review.
B
a literature review involves a review of research in which no statistical hypothesis tests were used.
C
a meta-analysis, but not a literature review, involves calculation of an effect size.
D
a literature review, but not a meta-analysis, involves calculation of an effect size.

A

The Correct Answer is “C”
Both meta-analyses and literature reviews involve a summation of research studies related to a particular topic or hypothesis. However, unlike a literature review, a meta-analysis involves calculation of an “effect size,” or a statistic that indicates the average effect of a particular treatment across the studies reviewed. This involves converting data from many different studies into a common metric so that their results can be combined and compared. A literature review, by contrast, provides descriptive information only.

127
Q

Your friend Bill has been involved in a motorcycle accident that resulted in a head injury. His neurologist
has indicated that Bill is experiencing aphasia. As a knowlegeable psychologist, you would give Bill all the following news about his injury except:
A
because his language loss is not severe, he has a better chance of recovery.
B
the greatest amount of recovery will occur in the next 3 months.
C
since he is right-handed he will probably exhibit milder aphasia.
D
he is lucky to be sixteen instead of sixty-six.

A

The Correct Answer is “C”
This is an extremely difficult question. That’s why you will review these tests several times. In regard to answer C, the opposite is true; Subirana (1969) found that left-handers generally exhibit milder aphasia, and recover more thoroughly and more quickly than right-handers. The course of recovery from aphasia due to head injury is related to several factors including age at time of injury–especially during the first three months (Kertesz, 1979).

128
Q

The most frequent complaints among individuals diagnosed with Undifferentiated Somatoform Disorder include:
A
Chronic fatigue, loss of appetite, gastrointestinal, or genitourinary symptoms
B
Joint pain, headache, or sexual indifference
C
Impaired coordination, paralysis, or double vision
D
Heart palpitations, chest pain, shortness of breath, or feeling of choking

A

The Correct Answer is “A”
According to DSM-IV-TR, Undifferentiated Somatoform Disorder requires the presence of one or more physical complaints that persist for 6 months or longer and which cannot be fully explained by another mental disorder or physical illness. Frequent complaints include chronic fatigue, loss of appetite, gastrointestinal, or genitourinary
symptoms. Choices B and C are more typically found in Somatization Disorder, which requires the presence of at least 8 different physical symptoms. Choice D includes symptoms of a panic attack.

129
Q
In psychotherapy, patients with Borderline Personality Disorder are most likely to exhibit:
A
feelings of inadequacy
B
idealization of the therapist
C
hostility towards the therapist
D
instability of thoughts and feelings about the therapist and themselves
A

The Correct Answer is “D”
This is one of those questions that requires you to select the best answer among several good choices. According to DSM-IV-TR, Borderline Personality Disorder is characterized by “A pervasive pattern of instability of interpersonal relationships, self-image, and affects, and marked impulsivity…” The other answer choices are characteristics typically seen among patients with BPD; however, Choice D offers the most general and complete description, and is, therefore, the best answer.

130
Q

Although conclusions will vary across different studies, which of the following statements is most supported by the overall body of research on therapy outcome for African-American and Caucasian patients?
A
There is no significant difference in outcome between African-American and Caucasian patients.
B
African-American patients have better outcomes overall than Caucasian patients.
C
Caucasian patients have better outcomes overall than African-American patients.
D
Treatment outcomes for African-American patients are better but only when the therapist is also African-American.

A

The Correct Answer is “A”
A. Questions like these are difficult to answer, because research in this area is contradictory, and the issue is not resolved. Research has clearly identified a number of variables that potentially interact with race in influencing therapy outcome. For instance, African-American patients tend to have poorer outcomes when working with therapists who are insensitive to or unknowledgeable about racial or cultural issues. There are also studies that show that African-Americans are more likely to terminate therapy prematurely than Caucasians, and even a few studies which show they are likely to have poorer outcomes. However, the bulk of the literature and thinking on this issue supports the notion that race, in and of itself, is not a good predictor of therapy outcome.

131
Q
Working Memory is an expansion of the WISC-III Freedom from Distractibility factor. It is comprised of which of the following scales?
A
Coding, symbol search, digit span
B
Arithmetic and digit-span
C
Perceptual organization, comprehension, similarities
D
Digit-span, picture arrangement, coding
A

The Correct Answer is “B”
The Working Memory scale is comprised of the WISC-III subtests of Arithmetic and Digit-Span and measures numerical ability, attention, and concentration.

132
Q
Which of the following sections of the 2002 Ethical Principles of Psychologists and Code of Conduct “discusses the intent, organization, procedural considerations, and scope of application of the Ethics Code.”
A
Introduction only
B
Introduction and Preamble
C
Preamble only
D
General Principles and Ethical Standards
A

The Correct Answer is “A”
A. The 2002 Ethics Code is divided into four sections: Introduction, Preamble, General Principles, and Ethical Standards. The Introduction “discusses the intent, organization, procedural considerations, and scope of application of the Ethics Code.” The purpose of both the Preamble and General Principles is to provide “aspirational goals to guide psychologists toward the highest ideals of psychology.” The Ethical Standards “set forth enforceable rules.”

133
Q
\_\_\_\_\_\_\_\_\_\_\_\_\_\_\_\_\_ is a present-oriented, structured and time-limited treatment that integrates biological and psychosocial approaches.
A
Motivational interviewing
B
Solution-focused therapy
C
Interpersonal psychotherapy
D
Reality therapy
A

The Correct Answer is “C”
C. Interpersonal psychotherapy (IPT) is a manualized psychotherapy first designed for the treatment of nonbipolar, nonpsychotic major depression with the focus on the problems of depression and interpersonal distress. IPT focuses on interpersonal aspects of depression and uses the biopsychosocial model which frames depression as a medical illness occurring in a social context The goal of IPT is to alleviate symptoms with interpersonal relationships as a point of intervention. IPT has been found to be effective for the treatment of depression patients from adolescence to late life, for women with postpartum depression and for patients with medical comorbidity (See: Weissman, M.M., Markowitz, J.W., Klerman, G.L. (2000). Comprehensive guide to interpersonal psychotherapy. New York, Basic Books.)

134
Q
According to M. Seligman's theory of learned optimism, a student with an optimistic attribution style who
fails an exam in a class which he usually does well in is most likely to say:
A
"I was unlucky"
B
"I didn't study enough"
C
"the teacher is always a tough grader"
D
"the test was hard this time"
A

The Correct Answer is “D”
In Seligman’s theory of learned optimism, attributions of optimistic people are believed to be the opposite of attributions of depressed people. Since depressed people make internal, stable, and global attributions to negative events, optimistic people would tend to make external, unstable, and specific attributions in response to negative events. Therefore, we can readily eliminate “B” (“I didn’t study enough”) since that’s an internal attribution. Choice “C” (“the teacher is always a tough grader”) is a stable attribution. That leaves Choices “A” (“I was unlucky”) and Choice “D” (“the test was hard this time”) – which are both external and unstable attributions. Of the two, however, Choice “D” is better since being unlucky would imply that success is a matter of luck.

135
Q
Components of health anxiety include disease conviction, disease fears, disease preoccupation, bodily checking and reassurance seeking, and disease-related avoidance and escape behaviors. Strong disease conviction is most associated with:
A
Specific (“disease”) Phobia
B
Delusional Disorder, Somantic Type
C
Hypochondriasis
D
Panic Disorder
A

The Correct Answer is “C”
C. Individuals with hypochondriasis have the presence of strong disease convictions, insisting that they have an undetected serious illness or disease. Often convictions result from misinterpreting normal bodily sensations and minor symptoms as serious disease warning signs. Specific “disease” phobia (a.) is a DSM-IV-TR Anxiety Disorder associated with a fear of acquiring or being exposed to a disease. It is also commonly a feature of hypochondriasis. An absence of disease conviction is a differential characteristic of the two disorders. The disease conviction for individuals with Delusional Disorder, Somatic Type (b.) reaches extremely strong, unreasonable and delusional proportions. Although during panic attacks people with panic disorder commonly worry about dying, disease conviction is not a characteristic that is strongly associated with Panic Disorder.

136
Q

According to Piaget, centration is:
A
characteristic of the formal operational stage and is the belief that others are as concerned with a subject’s behavior as the subject is.
B
characteristic of the preoperational stage and involves focusing on one aspect of a situation while neglecting others.
C
characteristic of the preoperational stage and is an inability to consider another’s point of view.
D
characteristic of the concrete operational stage in which it is understood that two objects can remain equal according to a certain measure despite changes to their perceived form.

A

The Correct Answer is “B”
According to Piaget, centration refers to a limitation of preoperational thought that leads a child to focus on one aspect of a situation and neglect others, often leading to illogical conclusions. Choice A describes an imaginary audience (a term coined by David Elkind). Choice C refers to egocentrism and Choice D describes conservation.

137
Q
A 10-year old child who has an IQ of 90 is having difficulty doing her homework because she doesn't make use of effective learning strategies. This child would benefit from training in
A
adaptive skills.
B
metacognitive skills.
C
mindfulness.
D
elaborative rehearsal.
A

The Correct Answer is “B”
Metacognition is the ability to monitor one’s own cognitive processes while thinking, learning, and remembering. It allows us to identify and use appropriate learning strategies, and the research has shown that learners with below-average intelligence can benefit from training in metacognitive skills. (Note that “mindfulness” – response C - is used in the educational psychology literature to refer to effort.)

138
Q
The ability to understand that changing an object's appearance doesn't change the object's physical qualities typically develops during years:
A
0 to 2
B
2 to 6
C
7 to 10
D
11 to 13
A

The Correct Answer is “C”
Conservation is the ability to understand that changing an object’s appearance doesn’t change the object’s physical qualities. For example, water poured from a tall thin glass into a short wide glass will be conserved, or understood to be the same amount of water. Conservation develops during the concrete operations stage (7 to 12 years). Note that there may be slight variations between age ranges that you have studied and the age ranges presented in test questions. In this case, the most correct choice is 7 to 10 years, which falls within the 7 to 12 year range. Choice D, 11 to 13 years, overlaps between the high end of concrete operations and the low end of formal operations (12 onward) – which is not as good a choice for the typical development of concrete operations

139
Q

Dr. Que has recently started working with a family consisting of a mother, father, and five year old boy. The parents say the boy has become “impossible to manage” and they don’t know what to do to control him. Dr. Que believes he will get a better idea of the family’s dynamics if he is able to observe them at home. When he suggests this, the parents say that, because of their religious beliefs, they cannot allow this. Dr. Que should:
A
refer the family to a therapist from the same religious background.
B
impress upon the parents his need to observe them at home in order to best serve the interests of the child.
C
determine if there is an alternative and mutually agreeable location for the observation.
D
contact child protective services immediately.

A

The Correct Answer is “C”
This type of situation is covered by ethical requirements related to cross-cultural counseling. (See, for example, APA’s Guidelines for Providers of Psychological Services to Ethnic, Linguistic, and Culturally Diverse Populations.) There’s certainly not any information given that warrants a call to child protective services. Also, it’s not necessary to make a referral, and it wouldn’t be a good idea to manipulate the parents into doing something they don’t want to do.

140
Q
Which of the following is an endogenous opiate?
A
dopamine
B
catecholamine
C
endorphins
D
heroin
A

The Correct Answer is “C”
Endorphins are endogenous (produced inside the body) peptides which activate opiate receptors in the brain and
block the release of Substance P, which results in decreased perception of pain. The term “endorphin” is an abbreviation of “endogenous morphine”. The only other opiate listed in the choices is heroin (“D”) which is derived from the opium poppy – an exogenous substance.

141
Q
Researchers are interested in detecting differential item functioning (DIF). Which method would not be used?
A
SIBTEST
B
Mantel-Haenszel
C
Lord’s chi-square
D
cluster analysis
A

The Correct Answer is “D”
In the context of item response theory, differential item functioning (DIF), or item bias analysis, refers to a difference in the probability of individuals from different subpopulations making a correct or positive response to an item, who are equal on the latent or underlying attribute measured by the test. The SIBTEST or simultaneous item bias test, Mantel-Haenszel, and Lord’s chi-square are statistical techniques used to identify DIF. Cluster analysis is a statistical technique used to develop a classification system or taxonomy. This method wouldn’t detect item bias or differences.

142
Q

Overall, research on the compressed work week suggests that its effects are:
A
increased job satisfaction, with no effect on job performance.
B
increased job performance, with no effect on job satisfaction.
C
decreased job performance, with no effect on satisfaction.
D
increased job performance and increased job satisfaction.

A

The Correct Answer is “A”
In the compressed work week (CWW), the number of days worked is shortened but the work hours per day are lengthened. For example, the work week might be compressed into four days a week, ten hours a day. Research results regarding the effects of the CWW are mixed, but most studies suggest that job satisfaction tends to increase, while job performance is not affected and (depending on the job) may even decrease. The CWW also tends to result in decreased absenteeism.

143
Q

A psychologist is asked by the attorney for the father in a divorce procedure to determine, in his expert opinion, which parent is better able to care for the children. The father is in the same state as the psychologist but the mother is in a distant state. The psychologist should
A
agree to evaluate the father and give his opinion as requested as long as the conclusions are not influenced by who pays the fee.
B
evaluate the father but not send the report until asked for it by the judge.
C
evaluate the children and both parents before giving such an opinion.
D
refuse to accept this work since there can be no firm predictions made as to which parent would be better able to care for the children.

A

The Correct Answer is “C”
The basic principle is that you can’t give an opinion as to which parent will be better able to care for the children unless you interview both parents and the children. Otherwise, all you can attest to is your findings about the psychological health of whomever you evaluate. You can’t, if you only see one of the parents, opine as to which one is better: You’d have no basis to come to such a conclusion.

144
Q

patient obtains a T-score of 100 on the MMPI-2’s F scale. This score indicates that
A
the examinee is trying to fake good.
B
the examinee is disoriented and confused.
C
the examinee is defensive.
D
the entire test should be considered invalid.

A

The Correct Answer is “D”
The F, or Infrequency, scale is one of the MMPI-2’s validity scales. It contains items that are answered in the scored direction very infrequently. Elevated scores on this score (i.e., T scores above 65) can occur for a number of reasons, including “faking bad,” confusion and disorientation, and cognitive deficits. Mildly to moderately elevated scores on the validity scales can be interpreted clinically; however, scores of about 90 or greater could indicate random responding and completely invalidate the test.

145
Q
According to Beck, depression is caused by
A
biochemical factors.
B
early parental rejection.
C
dysfunctional automatic thoughts.
D
dysfunctional irrational thoughts
A

The Correct Answer is “C”
The purpose of Beck’s cognitive therapy is to modify dysfunctional automatic thoughts that are seen as the cause of maladaptive emotional responding (e.g., depression and anxiety). Unlike Ellis, who identifies irrational thoughts as the cause of clients’ problems, Beck believes that these thoughts may or may not be irrational.

146
Q
The best treatment for someone who has difficulty making friends, is shy, and can't maintain steady employment is:
A
individual social skills training.
B
paradoxical intention.
C
token economy.
D
group social skills training.
A

The Correct Answer is “D”
Treatment for very shy individuals who have trouble in interpersonal settings (as implied by the fact that the person cannot hold a job) typically involves social skills and assertiveness training. Although both of these treatments can be administered in individual therapy, the group format has a number of advantages: there are multiple models, and opportunities for feedback, support, and vicarious learning. These aren’t available in individual training sessions. Thus, D is the best answer.

147
Q

A multivariate analysis of variance would be used to analyze collected data when:
A
the researcher wants to analyze the effects of an extraneous variable
B
the researcher wants to remove the effects of an extraneous variable
C
the study includes two or more independent variables
D
the study includes two or more dependent variables

A

The Correct Answer is “D”
The multivariate analysis of variance (MANOVA) is a type of ANOVA used when two or more dependent variables are included in a study. Rather than using separate ANOVAs to evaluate the effects of each of the dependent variables, a researcher could use the MANOVA when all the dependent variables are measured on a ratio or interval scale. This also helps to control the experiment-wise error rate.

148
Q
The Health Belief Model (HBM) is a tool for predicting and understanding an individual’s health-related decision making. The results from which dimension provide the most information?
A
perceived severity
B
perceived susceptibility
C
perceived benefits
D
perceived barriers
A

The Correct Answer is “D”
D. The Health Belief Model (HBM), which was developed initially to understand why people seemed unwilling to use preventative measures and screening tests available, is based on psychological and behavioral theory. The four dimensions the model considers in the influence of health-related decision making and behavior are: perceived susceptibility; perceived severity; perceived benefits; and perceived barriers. The dimension identified as the most influential variable for predicting and explaining health-related decision-making and actions is perceived barriers. The effectiveness of the action (screening, preventative care) and the perceptions of inconvenience, expense, unpleasantness, dangerousness, etc., are examples of perceived barriers.

149
Q
Which of the following methods of establishing a test's reliability is, all other things being equal, likely to be lowest?
A
split-half
B
Cronbach's alpha
C
alternate forms
D
test-retest
A

The Correct Answer is “C”
You probably remember that the alternate forms coefficient is considered by many to be the best reliability coefficient to use when practical (if you don’t, commit this factoid to memory now). Everything else being equal, it is also likely to have a lower magnitude than the other types of reliability coefficients. The reason for this is similar to the reason why it is considered the best one to use. To obtain an alternate forms coefficient, one must administer two forms of the same test to a group of examinees, and correlate scores on the two forms. The two forms of the test are administered at different times and (because they are different forms) contain different items or content. In other words, there are two sources of error (or factors that could lower the coefficient) for the alternate forms coefficient: the time interval and different content (in technical terms, these sources of error are referred to respectively as “time sampling” and “content sampling”). The alternate forms coefficient is considered the best reliability coefficient by many because, for it to be high, the test must demonstrate consistency across both a time interval and different content.

150
Q
Approximately what percent of women experience full-blown postpartum (clinical) depression?
A
1 to 5%.
B
10 to 20%.
C
20 to 30%.
D
30 to 40%.
A

The Correct Answer is “B”
The majority of women experience some depression following childbirth but, for most, these symptoms are mild. For about 10 to 20% of women, symptoms are sufficiently severe to qualify for a diagnosis of Major Depression.

151
Q
Which of the following individuals would agree that the acquisition of language is due largely to biologically-programmed capacities?
A
Piaget
B
Vygotsky
C
Chomsky
D
Bruner
A

The Correct Answer is “C”
The belief that language acquisition relies primarily on innate biological capacities is referred to as the nativist position. Of the theorists listed, only Chomsky is classified as a nativist. He proposed the existence of an innate “language acquisition device.”

152
Q
You conduct a study designed to assess the effectiveness of psychotherapy in the treatment of depression. You work with two groups, one of which receives the therapy and one of which is an attention-only control group. All of your subjects are hospitalized inpatients; thus, all of them are extremely depressed and therefore score extremely low on your pretest measure of depression. The biggest threat to external validity in this study is:
A
regression to the mean
B
reactivity
C
interaction between selection and treatment
D
pretest sensitization
A

The Correct Answer is “C”
Note that you are being asked for the biggest threat to external validity, not internal validity in this question. Therefore, you can rule out regression to the mean, which is generally viewed as a threat to internal validity (regression probably wouldn’t threaten internal validity anyway in this case, since both groups appear to be equivalent in terms of their baseline depression levels). External validity refers to the generalizability of research results. An “interaction between selection and treatment” means that the effect of a treatment may not generalize to other members of the target population who differ in some way from the research subjects. For example, in this case, it’s possible that your therapy is effective for individuals who are highly depressed, but would not have any effect on individuals who are moderately depressed.

153
Q

A psychiatrist sends you a release for information from a former patient of yours requesting records that are seven years old. In this situation, you should
A
send the records as requested.
B
send the records with a cover letter explaining the limitations of the records.
C
refuse to send the records.
D
contact the patient to make sure he or she understands the ramifications of the situation.

A

The Correct Answer is “B”
It is your responsibility to take steps to prevent the misuse of potentially obsolete data. At the same time, access to these records could facilitate the psychiatrist’s work with your client, and the client does have the right to request the records. Thus, a cover letter explaining the limitations of the records (i.e., they are old and probably obsolete at least to some degree) is the way to go here.

154
Q

Which of the following statements is MOST consistent with the research results on the effects of crowding?
A
Women are more adversely affected by crowding than men.
B
The effects of crowding are mediated by a person’s level of distraction.
C
Crowding always has a negative impact on individuals’ affect.
D
Individuals are more willing to discuss intimate topics in crowded situations vs. uncrowded situations.

A

The Correct Answer is “B”
The effects of crowding are mediated by a person’s level of distraction; for example, a person viewing a boring documentary in a crowded movie theater has been found to feel the effects of crowding more than a person
watching an interesting movie in the crowded theater. Contrary to “A,” men tend to be more adversely affected by crowding than women. Contrary to “C,” crowding does not always have a negative impact on affect; rather, it tends to enhance whatever a person is already feeling (e.g., a person at a crowded party or sports event may experience enhanced positive feelings). And common sense should have allowed you to eliminate “D” because people are less willing to discuss intimate details in crowded situations.

155
Q
The three dimensions of situational control identified by Fiedler include all of the following except:
A
leader-employee relations
B
position or legitimate power
C
role expectations
D
task structure
A

The Correct Answer is “C”
C. Fiedler’s Contingency Model states leaders have a dominant leadership style that is resistant to change and distinguishes between two types of leaders – high LPC leaders (person-oriented, more focused on maintaining good interpersonal relationships) and low LPC leaders (task-oriented, more focused on successful task performance). Changes in the structure of the situation can improve the chances of success as the leader’s success is contingent on the situation, task to be completed, leader’s style or personality, and the maturity of the group. Fielder proposes task-oriented (low-LPC) leaders are most effective when the leader has either low or high situational control and person-oriented (high-LPC leaders) are most effective when situational control is moderate. According to Fielder, situational control is determined by: leader-member relations, task-structure, and leader position power.

156
Q
Research comparing patients with Schizophrenia from non-Western developing countries to those from Western industrialized countries has found that they differ in terms of:
A
symptoms
B
age and gender
C
gender and prognosis
D
course and outcome
A

The Correct Answer is “D”
Several studies conducted by the World Health Organization have consistently found differences in the course and outcome of Schizophrenia patients from developing and industrialized countries. Patients from developing countries more often exhibit an acute onset of symptoms, a shorter clinical course, and a complete remission of symptoms. No consistent differences were found between these two groups in regards to age, gender, or type of symptoms.

157
Q

The police inform you that they have a warrant out on one of your clients who is suspected of several felony charges of grand theft auto. They want you to provide them with the client’s address and phone number. You should:
A
cooperate with the police
B
require them to get a warrant before you release any information
C
refuse to provide them with the information and notify your client
D
inform them when your next scheduled appointment is with your client

A

The Correct Answer is “C”
Since there is no indication that your client is in imminent danger to self or others, you would be obligated to protect the confidentiality rights of your client. However, you should notify your client of this information that you received, and discuss the option of turning himself in to the police.

158
Q

Equity theory suggests that the relationship between pay and motivation
A
is positive; the more one is paid, the more motivated one is likely to be.
B
is negative; the more one is paid, the less motivated one is likely to be.
C
is negligible; pay has little or nothing to do with motivation.
D
is complex; the relationship between pay and motivation is mediated by a variety of other factors.

A

The Correct Answer is “D”
Equity theory holds that, in any work situation we assess both our input (how much effort we are contributing to a work situation) and our output (how much we get out of it). We then compare our input/output ratio to what we perceive are the ratios of our co-workers. A state of balance exists when we perceive that our input/output ratios are equal to those of our co-workers. An unbalanced situation is one in which these ratios are not perceived as equal. For instance, underpayment is an unbalanced situation in which outcomes are perceived to be less than contributions, and overpayment occurs when we perceive that our outcomes exceed contributions. Equity theory holds that motivation is affected by these perceived input/output ratios. For instance, if you perceive that you are contributing more to a company than a co-worker who is getting paid the same amount, you would take action to correct this inequity, whether it be by working less, asking for a raise, leaving the job, re-evaluating the value of your contributions, etc. Similarly, if you are getting paid more than a co-worker whom you perceive as just as valuable as yourself, you may work more, do better quality work, decide that you are more important than the co-worker, etc. The point is that the theory suggests that the relationship between pay and motivation is complex and will be mediated by one’s perceived input, the perceived input/output ratio of co-workers, and the strategy chosen to achieve a state of balance or equity.

159
Q
The most effective behavioral technique to reduce a school-aged child’s temper tantrums is:
A
modeling
B
extinction
C
positive reinforcement
D
negative reinforcement
A

The Correct Answer is “B”
B. Extinction is the elimination or reduction in the frequency of a response achieved by the removal of the reinforcement maintaining the response. The extinction of temper tantrums would entail ignoring the child’s behavior or not reinforcing it with attention. This technique has been found to be effective for reducing temper tantrums. Positive (c.) and negative (d.) reinforcement are used to strengthen behaviors. Modeling (a.) is used to teach new behaviors.

160
Q
In Ainsworth's "strange situation," a child exhibits the disorganized/disoriented attachment pattern. This pattern is most associated with:
A
a difficult temperament.
B
mental retardation.
C
early maltreatment.
D
early enrollment in daycare.
A

The Correct Answer is “C”
The disorganized/disoriented attachment pattern was added to Ainsworth’s original three patterns by Mary Main, who found it to be a common pattern among children who had been mistreated by their caregivers.

161
Q

When a psychologist believes that her patient’s records will be used in a legal proceeding, she should:
A
rewrite them to meet the higher standards of a forensic setting
B
avoid the use of any psychological terminology in favor of legal terminology
C
maintain them in the same kind and quality as all of her patients’ records
D
maintain them in the kind and quality consistent with reasonable scrutiny in an adjudicative forum

A

The Correct Answer is “D”
According to the Specialty Guidelines for Forensic Psychologists (1991), “When forensic psychologists conduct an evaluation or engage in the treatment of a party to a legal proceeding, with foreknowledge that their professional services will be used in an adjudicative forum, they incur a special responsibility to provide the best documentation possible under the circumstances.”

162
Q
The ratio of Major Depressive Disorder in females to males is approximately
A
1:1.
B
2.5:1.
C
5:1.
D
1:2.
A

The Correct Answer is “B”
Estimates of the female to male Major Depression rate range from 2:1 to 3:1. Thus, 2.5:1 is the best answer to this question

163
Q
If a child can understand that A is greater than B and B is greater than C, and then understand that as a result, A is greater than C, this is an example of:
A
Inductive reasoning
B
Symbolic thought
C
Deductive reasoning
D
Pre-operational thought.
A

The Correct Answer is “A”
This is an example of inductive reasoning, or reasoning from a particular fact to a general rule. B, symbolic thought, is the understanding that one thing can stand for another, C, deductive reasoning, is reasoning from a general law to a particular case, and D is object permanence, the understanding that objets continue to exist even when they are not visible.

164
Q

During the 10th psychotherapy session with a client, you realize that your client has recently started dating your best friend. You should:
A
refer the client to another therapist
B
discuss this with your client immediately
C
discontinue your relationship with your friend
D
wait and address the problem with your client if a conflict becomes imminent

A

The Correct Answer is “B”
Although the Ethics Code does not specify exactly how to resolve this type of ethical dilemma, it does state, “If a psychologist finds that, due to unforeseen factors, a potentially harmful multiple relationship has arisen, the psychologist takes reasonable steps to resolve it with due regard for the best interests of the affected person and maximal compliance with the Ethics Code” (Ethical Standard 3.05[b]). Consistent with the intent of the Code, the best option would probably be to discuss the matter with your client immediately. After discussing it with your client, additional steps, such as referring the client, may be appropriate.

165
Q
The use of "pooled variance" in statistics assumes that:
A
the sample sizes are equal
B
the sample variances are equal
C
the population sizes are equal
D
the population variances are equal
A

The Correct Answer is “D”
Pooled variance is the weighted average variance for each group. They are “weighted” based on the number of subjects in each group. Use of a pooled variance assumes that the population variances are approximately the same, even though the sample variances differ.

166
Q

According to Marlatt’s theory of substance dependence:
A
a relapse of dependence is likely if the person makes non-dispositional attributions for use following a “slip” (use of a drug after a period of abstinence).
B
substance use is “over-learned” in that it is maintained by itself as well as by multiple cognitive mediators and external reinforcers.
C
addictions can be easily extinguished.
D
excessive substance use is related to an unresolved need for power.

A

The Correct Answer is “B”
Marlatt’s model of substance dependence holds that addicts learn to associate substance use with relief of self-criticism and guilt through a variety of cues and reinforcers, such as advertisements depicting people feeling cheerful when drinking and social occasions in which a carefree attitude is reinforced. In other words, there are a variety of cues and mediators that serve to encourage and reinforce use. Contrary to the person’s expectations, however, excessive substance use only exacerbates problems, such as interpersonal or work-related problems. This leads to more self-criticism and guilt, which the person again attempts to relieve by using. In other words, substance use is self-reinforcing – it is the cause of and the expected solution to the same problems. And it is “over-learned” in that, due to the multiplicity of its antecedents, it becomes a strongly ingrained behavior. You might have gone for choice A, since you probably knew that Marlatt is associated with a theory of relapse prevention that has to do with attributions regarding the reasons for “slips.” However, according to Marlatt, a slip is likely to lead to a full-blown relapse when the person makes dispositional attributions for it, such as when the person blames him or herself. Relapse prevention involves teaching the person to make non-dispositional
attributions, such as blaming the situation or the nature of the disease. So this question illustrates the importance of reading carefully and processing what you read, rather than relying on the recognition of “buzzwords.”

167
Q
According to Holland, a person who enjoys filing, copying, and data entry would have which of the following personality types?
A
realistic
B
investigative
C
conventional
D
obsessive-compulsive
A

The Correct Answer is “C”
In Holland’s personality typology, the Conventional type most enjoys manipulating data, filing, and copying. The Realistic type (A) prefers working with tools and machinery. The Investigative type (B) is analytical, precise, and curious. The Artistic type is creative, introspective, and nonconforming. The Social type enjoys working with others. And the Enterprising type seeks economic or professional achievement through manipulating others. Obsessive-compulsive (D) is not one of Holland’s personality types.

168
Q

A research study using an ABAB design involves the following elements in sequence:
A
treatment, baseline, treatment, baseline
B
baseline, treatment, baseline, treatment
C
baseline, intervention in setting A, baseline, intervention in setting B
D
baseline for group A, treatment for group A, baseline for group B, treatment for group B

A

The Correct Answer is “B”
An ABAB design is a type of single-subject design. It is also an example of a reversal design – a baseline measure of a behavior is obtained (the “A” phase), the behavior is again measured after a treatment is administered (the “B” phase), the treatment is removed or reversed and the behavior is again measured (the second “A”), and the behavior is again measured after the treatment is re-applied (the second “B”). Choices C and D are examples of multiple baseline designs. Multiple baseline designs do not involve withdrawal of treatment, but rather, apply the treatment to multiple settings (multiple baseline across settings) or to the same behavior of different subjects (multiple baseline across subjects).

169
Q
A whisper being audible in a library reading room, but not in a busy cafeteria is explained by:
A
the all-or-nothing principle
B
the “law of effect”
C
Weber’s law
D
the “law of proximity”
A

The Correct Answer is “C”
C. Weber’s law is one of the psychophysical laws that explains the relationship between physical stimuli and their psychological effects. According to Weber, the “just noticeable difference” in the stimulus is proportional to the magnitude of the original stimulus, explaining why a whisper can be heard in a quiet room but not in a noisy one.

170
Q

Which of the following statements is not contained in APA’s Ethical Principles of Psychologists and Code of Conduct?
A
“the purely private conduct of psychologists … is not within the purview of the Ethics Code.
B
“This Ethics Code … has as its goals the welfare and protection of the individuals and groups with whom psychologists work …”
C
“The fact that a given conduct is not specifically addressed by an Ethical Standard does not mean that it is necessarily either ethical or unethical.”
D
“A violation of the Ethics Code usually means … that a psychologist will be legally liable in a court action or face other legal consequences.”

A

The Correct Answer is “D”
To answer this question, you didn’t need to have had the Ethics Code memorized. All you needed was a good understanding of what the Code is all about. Choice D is not part of the Ethics Code, since ethical and legal standards, even though they sometimes overlap, are distinct, with the Ethics Code often holding psychologists to more stringent standards. In fact, the Code states that “whether or not a psychologist has violated the Ethics Code does not by itself determine whether he or she is legally liable in a court action.”

171
Q
Extinction treatment paradigms are often limited by:
A
improperly applied reinforcers.
B
accidental reinforcement.
C
client expectations.
D
practical constraints such as time.
A

The Correct Answer is “B”
In extinction interventions, you don’t want anything to accidentally reinforce the behavior. Let’s say you advised parents to ignore their child’s whining attention-seeking behavior, and the behavior reduced in intensity. Then, Grandma comes for a visit and when the child whines she gives him a cookie. This would be reinforcing, and so we’d be back to square zero and would have to do the extinction all over again. So, accidentally applied reinforcement often interrupts the extinction procedure.

172
Q

In an experiment, a psychologist establishes a conditioned startle response to a flashing red light by pairing presentation of the light with a loud noise that naturally elicits a startle reaction. Subsequently, the red light is
simultaneously presented with a strong odor just before the loud noise. After many of these conditioning trials, which of the following is likely to occur when the strong odor is presented alone?
A
The strong odor will produce a startle response that is even stronger than the response produced by the red light due to sensitization of the subject.
B
The strong odor will produce a very weak or no startle response due to habituation.
C
The strong odor will not produce a startle response because blocking will have occurred.
D
The strong odor may or may not produce a startle response depending on the original strength of the subject’s response to the loud noise.

A

The Correct Answer is “C”
Blocking occurs when a CS is presented simultaneously with a second stimulus just before the US. Although it would seem that the second stimulus should acquire the properties of a CS from this procedure, that’s not what happens. Instead, the second stimulus does not produce a conditioned response. This is referred to as blocking.

173
Q

In a “split-brain” patient,
A
the patient cannot name objects or comprehend information that is projected solely to the right hemisphere.
B
the patient cannot name objects or comprehend information that is projected solely to the left hemisphere.
C
both hemispheres could process linguistic information, even though language production is controlled by the left hemisphere.
D
both hemispheres could process linguistic information, even though language production is controlled by the right hemisphere.

A

The Correct Answer is “C”
A “split-brain” patient is someone whose corpus callosum (the bundle of fibers which connects the right and left
hemispheres of the cortex) has been severed. Since the left hemisphere controls language production, these patients are unable to name objects projected to their right hemisphere because the information can’t be transferred from the right to the left. However, research with these patients has demonstrated that the right hemisphere can comprehend some aspects of language.

174
Q
There is evidence that the effectiveness of systematic desensitization for reducing anxiety is actually due to repeated exposure to the feared stimulus, which leads to extinction of the anxiety response. However, systematic desensitization was originally developed as an application of
A
negative reinforcement.
B
counterconditioning.
C
stimulus discrimination.
D
avoidance conditioning
A

The Correct Answer is “B”
In systematic desensitization, anxiety-arousing stimuli are paired with stimuli that produce an incompatible response (often relaxation). In other words, it was designed to use counterconditioning in order to eliminate an anxiety response. Some research suggests, however, that it is actually just the exposure to anxiety-arousing stimuli, without aversive consequences, that explains the effectiveness of this technique.

175
Q

Compared to the Stanford-Binet, the WAIS-III tends to:
A
underestimate the IQ scores of higher functioning individuals.
B
underestimate the IQ scores of lower functioning individuals.
C
overestimate the IQ scores of lower functioning individuals and underestimate the IQ scores of higher functioning individuals.
D
underestimate the IQ scores of lower functioning individuals and overestimate the IQ scores of higher
functioning individuals.

A

The Correct Answer is “C”
The easiest way to deal with this question and other questions like it is to remember that for the extremes of the IQ continuum, the Stanford-Binet is a better measure to use. It will more accurately reflect the person’s functioning at either the very top or very bottom of the range.

176
Q

The best way to increase the intra-rater reliability of a test that is subjectively scored would be to
A
train raters to pay very close attention to the scoring of the test.
B
have a second set of raters rescore the test.
C
use mutually exclusive and exhaustive rating categories.
D
ensure that the rating instrument is correlated with multiple criterion measures.

A

The Correct Answer is “A”
Note that this question is asking about intra-rater reliability, not inter-rater reliability. The prefix “intra” means the same thing it means in other contexts (e.g., intra-psychic): inside or internal. Thus, the question is asking you how to increase the internal consistency of a rating instrument. Of the choices listed, the only one which makes sense is to train the raters to pay more attention to what they are doing. If they pay closer attention, they are less likely to score the test inconsistently. Incidentally, mutually exclusive and exhaustive rating categories (choice C) are useful for increasing the inter-rater reliability of a test.

177
Q

Which of the following is not an advantage of the use of co-therapists in group therapy, according to Yalom?
A
The observational range of co-therapists is greater than that of a single group leader.
B
Co-therapists broaden the range of possible transferential reactions, making the nature of the client’s transferential distortions more evident.
C
A co-therapist can provide a beginning group therapist with needed objectivity and expertise.
D
When co-therapists pursue their own separate agendas in therapy, clients have the opportunity to intervene and learn conflict management skills.

A

The Correct Answer is “D”
According to Yalom, a potential disadvantage of the co-therapy format is that co-therapists may be overly competitive and pursue their own interpretations rather than supporting inquiries begun by the group. If this occurs, the group will be distracted and unsettled. The other choices describe potential advantages of the use of co-therapists.

178
Q

When comparing feminist therapy to a more psychodynamic approach, feminist theory
A
Disallows the mother’s role
B
Takes factors other than a woman’s view of herself into consideration
C
Relies much more on the emphasis of the biological mother
D
Encourages women to call men to task for the oppression of women

A

The Correct Answer is “B”
The feminist approach to psychotherapy rests on the assumption that social roles and socialization are important determinants of behavior. Psychological conflicts are then alternative roles and options. A primary goal is empowerment or helping women become more self-defining and self-determining.

179
Q

To reduce a child’s aggressive behavior, the best approach is to:
A
punish the aggression consistently and harshly.
B
use differential reinforcement of alternative behaviors.
C
identify the consequences of the behavior and alternatives to it.
D
have the child vent his or her aggressive feelings by hitting a pillow.

A

The Correct Answer is “C”
There are a variety of approaches to the treatment of aggression in children; the cognitive-behavioral approach is probably most common. In children who are older or developmentally advanced enough to understand, cognitive approaches tend to focus on helping the person accurately interpret external cues, so that he or she does not respond inappropriately with aggression. In younger children, the goal is often to identify the child’s goals, the negative consequences of using aggression to meet those goals, and alternatives to aggression. The other choices are not as likely to be helpful: consistent discipline is good, but contrary to choice A, overly harsh discipline can serve to increase the child’s aggression. Reinforcement for alternative behaviors is not a proven method of directly addressing aggression. And the notion that aggression is something that can be released through venting it on a safe target (or, by the way, watching it on TV) has not been supported – apparently, engaging in aggressive behavior only leads to more of the same.

180
Q
The Personality Disorder that has been found to have the best prognosis is:
A
Borderline Personality Disorder
B
Paranoid Personality Disorder
C
Dependent Personality Disorder
D
Obsessive-Compulsive Personality Disorder
A

The Correct Answer is “A”
A. The majority of individuals diagnosed with Borderline Personality Disorder (BPD) demonstrate significant reduction or remission of symptoms by middle age or sooner. Longitudinal studies involving those diagnosed in adolescence or early adulthood indicate that symptom resolution with impulsive symptoms are the quickest to
resolve, followed by cognitive and interpersonal symptoms and finally affective symptoms are the most chronic and show the least improvement with increasing age.

181
Q

Differences in communication style can act as a barrier in cross-cultural counseling. For example, some cultures have a high-context style, while others are more low-context. Which of the following is true about high-context communication and culture?
A
The communication found in high-context cultures reflects an emphasis on rules of law and procedures.
B
People belonging to high-context cultures rely heavily on nonverbal cues when communicating with others.
C
People in high-context cultures make extensive use of elaborative codes in their speech.
D
High-context communication changes rapidly and easily.

A

The Correct Answer is “B”
The notions of high- and low-context communication were introduced by Edward T. Hall, the author of The Silent Language (1959) and The Hidden Dimension (1969). In the United States, the communication style of the Anglo majority is low-context, but many racial/ethnic minorities (e.g., Latino, Asian, African-American) make use of a high-context style. High-context communication is based on nonverbal language. Low-context communication on the other hand is based upon the verbal part of messages.

182
Q

Bob, who came from a poor family, was encouraged throughout high school to take vocational classes due to his “low aptitude” scores. Bob ignored this advice and focused on college preparatory courses. After graduating Magna Cum Laude from Harvard Business School, he wrote his old alma mater and asked for a complete copy of his records. Bob’s grandmother said, “While you are at it–tell them to burn mine”. According to the Buckley Amendment,
A
Overall, the school must comply with both requests
B
They need to honor Bob’s wishes but not his grandmother’s
C
They need to honor Bob’s grandmother’s request, but not Bob’s
D
Then can disregard both requests

A

The Correct Answer is “A”
This question is a little tricky. While they don’t literally have to “burn” the records of Bob’s grandmother, overall both requests should be honored. According to the Buckely Amendment or the Family Educational Rights and Privacy Act, eligible students after age eighteen and their parents have the right of access to their children’s educational records and can challenge any content thereof. Also records that are no longer useful or relevant for the students or the educational institution are to be destroyed. Although Bob would not have any legal rights regarding the disposition of his grandmother’s records, once the school is made aware of the obsolete records they should destroy them.

183
Q
You are asked to evaluate a 3-year old child who has no interest in peer relationships, engages in highly unusual play behavior on his own, and is preoccupied with parts of objects. The child has no cognitive or language impairments. Based on this information, the correct diagnosis for this child is
A
Asperger's Disorder.
B
Rett's Disorder.
C
Autistic Disorder.
D
Schizoid Personality Disorder.
A

The Correct Answer is “A”
Asperger’s Disorder is a pervasive developmental disorder characterized by qualitative impairment in social interaction and restricted, repetitive, and stereotyped patterns of behaviors, interests, and activities. By definition, there are no clinically significant general delays in cognitive and language development. Of the choices listed, it is the diagnosis that best matches the child’s symptoms. In Rett’s Disorder, there are physical symptoms (e.g., deceleration of head growth) that are not mentioned in the question. In Autistic Disorder, there are significant impairments in language development. And in Schizoid Personality Disorder, stereotyped behavior and interests would not be present.

184
Q
Specific Phobia, Blood-Injection-Injury Type is characterized by each of the following EXCEPT:
A
a strong vasogal response
B
onset in late adolescence
C
more common in women than men
D
strong familial patterns
A

The Correct Answer is “B”
Specific Phobia, Blood-Injection-Injury Type, sometimes referred to as health phobia, is described in the DSM-IV-TR as “specified if the fear is cued by seeing blood or an injury or by receiving and injection or other invasive medical procedure. This subtype is highly familial (d.) and is often characterized by a strong vasogal response (a.).” The age of onset is usually early childhood and may occur at a younger age for women than men. In contrast to the other Specific Phobias’ usual physiological response of acceleration of heart rate and elevation in blood pressure, 75% of individuals with Specific Phobia, Blood-Injection-Injury Type report a history of a vasogal fainting response. The physiological response involves a brief acceleration of heart rate and elevation in blood-pressure followed by a deceleration and drop in blood pressure.

185
Q

Someone advocating an emic rather than an etic approach to cross-cultural psychology would argue that the study of a culture
A
should be undertaken from a detached and objective position.
B
should be undertaken from within the culture itself.
C
should involve a comparison across different cultures.
D
should use the history of the culture being studied as a reference point.

A

The Correct Answer is “B”
The emic/etic distinction was first made by an anthropologist named Pike in 1954 and has since been applied to cross-cultural psychology. The etic approach to the study of a culture involves studying it from the outside, using universally accepted means of investigation. The emic approach, by contrast, involves studying the culture from the inside and trying see it as its own members do.

186
Q
Which of the following medications would MOST likely cause confusion?
A
fluoxetine
B
amitriptyline
C
sertraline
D
paroxetine
A

The Correct Answer is “B”
Compared to the SSRIs, the tricyclics are much more likely to cause anticholinergic effects. Confusion is one of several possible anticholinergic effects. Other anticholinergic effects include dry mouth, blurred vision, constipation, urinary retention and tachycardia. The only anticholinergic listed in this question is amitriptyline. The other three choices are SSRIs: fluoxetine (Prozac), sertraline (Zoloft), and paroxetine (Paxil).

187
Q
On the WISC-III, "freedom from distractibility" is measured using which of the following subtests?
A
Digit Span and Digit Symbol
B
Arithmetic and Digit Span
C
Picture Completion and Picture Arrangement
D
Coding and Arithmetic
A

The Correct Answer is “B”
The freedom from distraction factor measures distractibility, concentration, and short-term memory. On the WISC-III, this factor is assessed with the Arithmetic and Digit Span subtests.

188
Q
The person who is unable to recognize certain objects when she grasps them and is unaware of that deficit is suffering from
A
agnosia.
B
ataxia.
C
aphasia.
D
akinesia.
A

The Correct Answer is “A”
Agnosia is the inability to identify something. It could be not recognizing a familiar face, a tone or sound, or not recognizing a body part when touched. If the incomprehension comes from language deficits, the usual term is aphasia. So for non-language comprehension deficits, the common term is agnosia. For instance, individuals with damage to the parietal lobe are sometimes unable to recognize objects touched with the contralateral hand. This condition is known as tactile agnosia. Ataxia refers most often to lack of muscle coordination. Akinesia refers to lack of voluntary motion, such as what occurs in some forms of catatonia.

189
Q

An adolescent exhibiting foreclosure is most likely to respond in which of the following ways when asked about her career choice?
A
I haven’t given my career much thought yet.
B
My father, who’s a lawyer, has convinced me that I’d make a good lawyer too.
C
I’ve given that a lot of thought and I think being a veterinarian best suits my interests.
D
That’s something that’s been bothering me lately and I’m worried that I’m never going to be able to reach a decision.

A

The Correct Answer is “B”
James Marcia distinguishes between four identity statuses. Identity foreclosure is characterized by a commitment to an identity (e.g., career) that was defined or suggested by a parent of other significant person.

190
Q

Which of the following statements is most reflective of research findings?
A
Both male and female teachers are more likely to criticize boys than girls.
B
Male teachers are more likely to criticize boys, while female teachers are more likely to criticize girls.
C
Female teachers are more likely to criticize boys, while male teachers are more likely to criticize girls.
D
Boys and girls are equally likely to be criticized by both male and female teachers.

A

The Correct Answer is “A”
Research has suggested that overall, boys receive more criticism than girls from both male and female teachers. Some research suggests that this pattern may be due mostly (if not completely) to the behavior of boys and girls in the classroom. For instance, some studies show that girls on the average are more successfully adjusted to the student role than boys; this is a primary reason why girls are less likely to be criticized. Boys, on the other hand, misbehave more often and more disruptively than girls in the classroom.

191
Q

The most correct statement that can be made about biofeedback as a therapeutic treatment for pain reduction is that
A
the significant effects found can be accounted for by the type of instrumentation employed.
B
it is effective for certain types of patients only.
C
its effectiveness has diminished over the years.
D
it is probably no different from relaxation training in its effectiveness

A

The Correct Answer is “D”
Research on the efficacy of biofeedback for pain reduction indicates that it is as effective as treatments such as relaxation training. This is true for the treatment of most disorders or conditions, however in cases such as the treatment of Raynaud’s Disease or migraine headaches, some studies suggest that biofeedback is the treatment of choice.

192
Q
In the multitrait-multimethod matrix, a low heterotrait-monomethod coefficient would indicate:
A
low convergent validity.
B
low divergent validity.
C
high convergent validity.
D
high divergent validity.
A

The Correct Answer is “D”
Use of a multitrait-multimethod matrix is one method of assessing a test’s construct validity. The matrix contains correlations among different tests that measure both the same and different traits using similar and different methodologies. The heterotrait-monomethod coefficient, one of the correlation coefficients that would appear on this matrix, reflects the correlation between two tests that measure different traits using similar methods. An example might be the correlation between a test of depression based on self-report data and a test of anxiety also based on self-report data. If a test has good divergent validity, this correlation would be low. Divergent validity is the degree to which a test has a low correlation with other tests that do not measure the same construct. Using the above example, a test of depression would have good divergent validity if it had a low correlation with other tests that purportedly measure different traits, such as anxiety. This would be evidence that the depression test is not measuring traits that are unrelated to depression.

193
Q

The primary purpose of feedback in the context of organizational development is to:
A
help clients understand the diagnostic information that has been collected
B
provide clients with information on the effectiveness of an OD intervention
C
provide employees with information about their individual performance
D
provide managers with information on the employee’s concerns

A
The Correct Answer is "A"
Organization development (OD) is a process used to facilitate organizational change. The phases of OD have been described by Burke as: entry, contracting, diagnosis, feedback, planning change, intervention, and evaluation. The purpose of the feedback phase is to help clients understand the information that the consultant has gathered and diagnosed so the clients can decide what actions to take
194
Q
Wolfgang Kohler, a founder of the gestalt school of learning and thought, is known for applying gestalt learning principles to the development of
A
unfinished business.
B
insight.
C
integration of all parts of the psyche.
D
awareness of psychological boundaries
A

The Correct Answer is “B”
Kohler, a founder of the gestalt school of psychology, studied insight learning. In one of his experiments, a
chimpanzee had to get some food placed outside his cage. He had two sticks but they were too short to reach the food. As he was sitting with the two sticks in his hand, the chimp had what Kohler called an “a-ha experience”: He quickly fit the sticks together and was able to get the food. This a-ha experience – a sudden, novel solution to the problem – is the basis of insight learning. According to Kohler, insight learning cannot be explained by traditional behavioral theories of learning, which are based on reinforcement and associations between stimuli.

195
Q
During the first of structural family therapy’s three steps, which of the following techniques is most useful?
A
constructing a family map
B
relabeling and reframing
C
enactment
D
tracking and mimesis
A

The Correct Answer is “D”
D. Structural family therapy entails three overlapping steps: joining, evaluating/diagnosing, and restructuring. Joining is the initial step in structural family therapy. Tracking (identifying and using the family’s values, life themes, etc.) and mimesis (adopting the family’s behavioral and affective style) are methods used to join the family system. Constructing a family map (a.) is a technique used for the structural diagnosis of the family. Relabeling and reframing (b.) are restructuring techniques. Enactment (c.) is used to facilitate diagnosis and restructuring of the family.

196
Q
Erikson proposed that psychosocial development continues throughout the lifespan. Successful resolution of the conflict of the final stage of development results in:
A
formation of intimate relationships.
B
participation in activities that promote the welfare of future generations.
C
development of mature ego defenses.
D
development of a sense of meaning.
A

The Correct Answer is “D”
The final stage in Erikson’s theory of psychosocial development occurs in late adulthood and involves a conflict between integrity and despair. Successful resolution occurs when the individual gains “wisdom” and finds meaning in life.

197
Q
Ultrasound techniques are least effective for detecting which type of traumatic brain injury?
A
shearing
B
tissue bruising
C
swelling
D
inter-cranial pressure (ICP)
A

The Correct Answer is “A”
A. Traumatic (physical) brain injuries are generally divided into three main types. The two more dangerous types are tissue bruising (b) and swelling (c), which cause tissues to swell and become compressed within the skull resulting in intercranial pressure (ICP). Shearing, or nerve fiber tearing, occurs from sudden impact of the brain with an object. Unlike tissue bruising and brain swelling, brain injury from nerve tearing is difficult to detect with ultrasound techniques such as x-rays, CT scans and MRIs which are most frequently used in traumatic brain injury cases. A PET (Positron Emission Tomography) scan is more effective in the detection of nerve tearing injuries. A PET scan measures metabolic processes, thus allowing an appraisal of how the brain is functioning. It tracks natural compounds, such as glucose, as the brain metabolizes them. By showing the areas of different metabolic activity, it then becomes easier to make diagnoses, such as determining the areas responsible for epileptic seizures.

198
Q

As an organizational psychologist, Dr. Jobb will recommend frame-of-reference training in order to
A
improve the accuracy of selection decisions.
B
enhance communication among team members.
C
reduce rater biases on performance appraisals.
D
improve leadership effectiveness

A

The Correct Answer is “C”
This is one of those concepts that you either know or don’t know, so it wouldn’t be worth a lot of time trying to figure out the right answer. Now you know: Frame-of-reference training is used to improve rater accuracy by teaching raters to focus on the various characteristics and requirements that contribute to good job performance.

199
Q
What is the approximate probability that a person with a Schizophrenic brother (not an identical or fraternal twin) would also have Schizophrenia?
A
60%
B
45%
C
10%
D
0.5%
A
What is the approximate probability that a person with a Schizophrenic brother (not an identical or fraternal twin) would also have Schizophrenia?
A
60%
B
45%
C
10%
D
0.5%